Infection MCQ
Infection MCQ
Infection MCQ
1. A biopsy from a patient who presented to your clinic following an eruption of pruritic papules revealed
lichenoid dermatitis with basal layer vacuolar degeneration, several necrotic keratinocytes, and some
extravasated red blood cells. There was some parakeratosis within the stratum corneum numerous eosinophils
in the dermis. Which diagnosis is most consistent with these findings?
A. Lichenoid drug eruption
B. Erosive lichen planus
C. Cicatricial pemphigoid
D. Aphthous stomatitis
E. Morsicatio buccarum
ANSWER: A.
EXPLANATIONS:
A. TRUE: The pathologic description is of lichenoid drug eruption. Numerous drugs and metals have been
implicated in lichenoid drug eruption (See Bologna 3rd Edition Table 11.2 in Chapter 11 for a comprehensive list).
What is highlighted is the presence of dermal eosinophils, which are more commonly seen histopathologically in
lichenoid drug eruptions when compared to lichen planus.
B. FALSE: Lichen planus (LP) could be an equally valid consideration based on the pathologic presentation but
would not exhibit the described features. While parakeratosis can be seen in mucosal lesions of LP, it is less often
seen in cutaneous lesions of LP. Additionally, the presence of numerous eosinophils should suggest alternative
diagnoses such as lichenoid drug eruption. The difference between LP and lichenoid drug eruption is also
highlighted in Chapter 11 of Bologna’s 3rd Edition page 194, Table 11-4).
C, D, and E, FALSE: These do not match the clinical depiction or the histopathologic description.
ANSWER: B.
EXPLANATIONS:
B. TRUE: The characteristic rash of smallpox consists of papules, vesicles, and pustules. Many lesions will have
central umbilications and will continue to crust over a 2-week period. Unlike Varicella, the lesions are all in the
same stage of development.
A. FALSE: The incubation period of smallpox is approximately 12 days.
C. FALSE: The smallpox virus is an orthopoxvirus.
D. FALSE: Guarnieri bodies are seen in all poxvirus infections and are not specific to smallpox.
E. FALSE: The most common complications of smallpox involve the respiratory system.
1
3. A 48-year-old African immigrant presents to the clinic with a flat-topped thin nodular ulceration on his dorsal
foot. A thin white thread-like object can be seen emerging from the center of the nodular ulceration. The man
notes recent intense burning pain in that leg. Which of the following is incorrect concerning this condition?
A. There is a 1-month time course from infection to symptoms.
B. The offending organism completes its lifecycle in water with copepods as the main vector.
C. Due to public health efforts, the incidence has decreased significantly over the last few
years.
D. This condition is caused by Dracunculus medinensis.
E. The main treatment is manual extraction along with public health prevention measures.
ANSWER: A.
EXPLANATIONS:
A. FALSE: This is the false statement. The vignette describes a man with dracunculiasis. The time course from ingestion
to symptoms is typically 1 year.
B. TRUE: When an infected individual with an exposed female worm steps into water, the female releases larvae into
the water, which make their way into freshwater copepods that are ingested by individuals who drink contaminated
water.
C. TRUE:In many African countries, the incidence of disease has decreased presumably due to more intense public
health measures.
D. TRUE: D. medinensis is the causative organism.
E. TRUE: While medications can be used, they do not typically eradicate the organism. It must be extracted manually.
Public health measures such as water source management and education are the principal means of preventing
disease.
4. Numerous tests to detect syphilis have been developed. Which of the following serologic tests is paired with the
proper corresponding description?
A. VDRL – Detects infection 1 week after infection and becomes non-reactive after treatment
B. RPR – Continues to stay reactive through late-stage syphilis and after treatment
C. ELISA – Has the lowest specificity of the serologic tests
D. MHA-TP – Becomes non-reactive after treatment
E. Fluorescent treponemal antibody absorption (FTA-ABS) – Becomes positive 3 weeks after
infection and is the most sensitive serologic test for primary syphilis
ANSWER: E.
EXPLANATIONS:
E. TRUE: FTA-ABS becomes positive 3 weeks after an individual is infected, and it is the most sensitive serologic test
of those listed for detecting primary syphilis.
A. FALSE: While VDRL does become non-reactive after treatment, it does not typically detect syphilis infection until
4 to 5 weeks after an individual is exposed to T. pallidum.
B. FALSE: RPR, like VDRL, becomes non-reactive in late-stage syphilis and in treated syphilis.
C. FALSE: ELISA possesses the highest specificity at nearly 100%.
D. FALSE: MHA-TP remains reactive after treatment, as does FTA-ABS.the most sensitive serologic test for primary
syphilis
2
5. Which of the following sets of clinical findings and infectious etiologies is incorrectly paired?
A. Grouped vesicles on the lower labial mucosa and herpes simplex virus I
B. Petechial and red macules on the soft palate and German measles
C. Painful vesicles and erosions on the posterior pharynx and echovirus
D. Asymptomatic gray-white papules on the buccal mucosa and cytomegalovirus
E. Red papules on the uvula and human herpesvirus 6 and 7
ANSWER: D.
EXPLANATIONS:
D. TRUE: Asymptomatic gray-white papules on the buccal mucosa are consistent with Koplik spots associated with
rubeola/measles and not cytomegalovirus. This viral enanthem appears during the prodromal phase consisting of
a fever, a cough, rhinorrhea, and rhinoconjunctivitis. At the time of the typical cephalocaudally spread exanthem,
the oral findings are no longer present.
A. FALSE: This describes findings consistent with orolabial herpetic gingivostomatitis, which can be caused by HSV
1 (more frequently) or 2.
B. FALSE: This represents Forscheimer spots associated with German measles, which is also known as rubella. This
enanthem is also typically asymptomatic and, like Koplik spots associated with measles, can easily be missed.
C. FALSE: This describes herpangina associated with enteroviruses such as coxsackievirus A and B, as well as
echovirus. These findings are less subtle and of concern in patients who may deprive themselves of adequate fluid
and nutrition intake associated with the severe pain from these lesions.
E. FALSE: This represents findings consistent with viral infection by human herpesvirus 6 and 7, which cause roseola
infantum (also called sixth disease). The oral findings are called Nagayama spots and are asymptomatic. However,
uvula and palatoglossal junctional ulcers are also associated with HHV6 and 7 and are a characteristic
symptomatic finding in patients with roseola infantum.
6. Which of the following parasites is incorrectly matched with the disease it causes and the typical treatment
offered to affected patients in the United States?
A. Lymphatic filariasis - Mansonella perstans - diethylcarbamazine
B. Loiasis - Loa loa - diethylcarbamazine
C. Strongyloidiasis - Strongyloides stercoralis - ivermectin
D. Onchocerciasis - Onchocerca volvulus - ivermectin
E. Amoebiasis - Entamoeba histolytica – metronidazole
ANSWER: A.
EXPLANATIONS:
A. TRUE: This is incorrectly matched. Lymphatic filariasis is caused by B. malayi, B. timori, or W. bancrofti. M.
perstans causes serous cavity filariasis. Diethylcarbamazine is the treatment of choice for lymphatic filariasis.
There is no current consensus on treatment for M. perstans serous cavity filariasis, but combination drug therapies
are being investigated.
B. FALSE: Loa Loa causes Loiasis. Diethylcarbamazine is the treatment of choice.
C. FALSE: Strongyloidiasis is caused by S. stercoralis. The treatment of choice is ivermectin.
D. FALSE: Onchocerciasis is caused by O. volvulus. The treatment of choice is ivermectin.
E. FALSE: Amoebiasis is caused by E. histolytica and is treated with metronidazole.
3
7. Which of the following statements is true concerning rhinoscleroma?
A. Chloramphenicol is bactericidal.
B. Histiocytes with engulfed bacteria known as Russell bodies are characteristic on histology.
C. It typically progresses through two stages: granulomatous and sclerotic.
D. Tetracycline and ciprofloxacin are typically used to treat the condition.
E. The causative organism is a gram-positive motile rod.
ANSWER: D.
EXPLANATIONS:
D. TRUE: Rhinoscleroma is a cutaneous infection caused by the bacterium Klebsiella rhinoscleromatis. Tetracycline
and ciprofloxacin are often used to treat rhinoscleroma.
A. FALSE: Chloramphenicol inhibits isolates, but it is not bactericidal.
B. FALSE: The histiocytes with engulfed organisms seen on histology are known as Mikulicz cells. Russell bodies are
eosinophilic immunoglobulin-containing inclusions within plasma cells that undergoing excessive synthesis of
immunoglobulin. Russell bodies are an uncommon finding often associated with chronic inflammatory conditions.
They are considered to be aggregated unreleased immunoglobulin components as a result of a block in the normal
secretion pathway of immunoglobulins. Intracytoplasmic Russell bodies may be seen in inflammatory processes
with a polyclonal immunoreactive pattern of plasma cells or in the context of plasmacytoma or multiple myeloma.
C. FALSE: There are typically three stages of rhinoscleroma: Atrophic, granulomatous, and sclerotic.
E. FALSE: Klebsiella rhinoscleromatis is a gram-negative non-motile rod.
8. A 46-year-old HIV-positive woman presents with tan-to-pink palmar macules. On further questioning, she
reports a painless ulcer on her labia a few months ago. The lesion resolved, and she thought nothing more of it.
The practitioner is worried that the patient may have secondary syphilis. Which of the following statements is
incorrect concerning syphilis treatment for this patient?
A. Primary syphilis in a non-HIV-infected patient is treated with a single IM dose of 2.4 million
units of benzathine penicillin G.
B. Congenital syphilis should be treated with aqueous crystalline penicillin G at 100,000–
150,000 units/kg/day for 10 days.
C. HIV-positive patients with neurosyphilis should receive 18-24 million units of aqueous
crystalline penicillin G per day for 10 to 14 days.
D. Early latent syphilis of unknown duration in an HIV-infected patient should be treated with
benzathine penicillin G 2.4 million units weekly for 3 weeks.
E. The Jarisch-Herxheimer reaction is caused by the release of TNF-alpha when spirochetes are
engulfed after antibiotic treatment.
ANSWER: D.
EXPLANATIONS:
D. TRUE: This is an incorrect statement. All early latent syphilis, regardless of duration, should be treated with a
single dose of 2.4 million units of benzathine penicillin G.
A. FALSE: Primary syphilis in HIV-infected patients (and non-HIV-infected patients) is treated with a single IM dose
of 2.4 million units of benzathine penicillin G.
B. FALSE: Congenital syphilis should be treated with aqueous crystalline penicillin G at 100,000 to 150,000
units/kg/day for 10 days.
C. FALSE: The regimen of treatment for neurosyphilis in HIV-positive patients is 18 to 24 million units of aqueous
crystalline penicillin per day for 10 to 14 days.
4
E. FALSE: The Jarisch-Herxheimer reaction is caused by the release of LPS and subsequent increased levels of TNF-
alpha cytokines when spirochetes are engulfed by phagocytes after antibiotic treatment. The symptoms mimic
bacterial sepsis.
9. A physician calls you about a patient in active labor, who was admitted earlier that morning. Upon admission,
the patient was documented as having a varicella infection which was acquired 2 days prior to admission, which
was confirmed by laboratory analysis. The physician is concerned about effects on the newborn.
Which of the following recommendations is most appropriate?
A. There are no further recommendations at this time.
B. The mother should be started on intravenous acyclovir as soon as possible.
C. The newborn should receive varicella zoster immunoglobulin as soon as possible.
D. The newborn should receive the varicella zoster vaccine as soon as possible.
E. The newborn should receive IV acyclovir as soon as possible.
Correct Answer: C. The newborn should receive varicella zoster immunoglobulin as soon as possible.
EXPLANATIONS:
Neonates born to mothers who have acquired varicella from 5 days prior to delivery until 2 days after birth require
varicella zoster immunoglobulin (VZIG) as soon as it can be administered following the delivery. Additional
precautions should be made to prevent infectious spread to other newborns while the newborn is hospitalized.
Incorrect Answers:
B. Treating the mother with intravenous (IV) acyclovir at this point would not affect the outcome or alter the risk of
the newborn developing neonatal varicella.
D. The varicella zoster vaccine is a live attenuated virus and should not be administered to newborns.
E. The administration of IV acyclovir to neonates born to mothers with varicella infection 5 days prior and 2 days
following delivery is not required. IV acyclovir should be initiated if newborns develop skin lesions or signs of
infection.
10. A young patient with diffusely distributed, flat, verrucous papules is found to have a mutation in the EVER1
gene. How did the patient most likely acquire this gene mutation?
A. Viral infection
B. Post-zygotic mutation
C. Autosomal recessive inheritance
D. Autosomal dominant inheritance
E. X-linked recessive inheritance
ANSWER: C.
EXPLANATIONS:
C. TRUE: EV most commonly has an autosomal recessive pattern of inheritance. HPV-5 and HPV-8 are associated
with EV-associated skin cancers but not the development of EV itself.
11. What is the most common presentation of herpes simplex virus infection in childhood?
A. Herpetic whitlow
B. Labial herpes
C. Genital herpes
D. Gingivostomatitis
E. Eczema herpeticum
5
ANSWER: D.
EXPLANATIONS:
D. TRUE: HSV-1 associated gingivostomatitis is the most common childhood presentation of the herpes virus.
Patients are often symptomatic only with skin lesions, but fevers, drooling, submandibular adenopathy, and
inflammation of the oral mucous membranes can occur. Occasionally, lesions can extend from the oral mucosa to
the lips, chin, and cheeks. Severe lesions such as ulcers can be painful enough to inhibit the necessary hydration
and nutritional demands of a child. In these cases, hospitalization is required for intravenous support.
A. FALSE: HSV infection of the digital pulp (herpetic whitlow) is also less common than gingivostomatitis.
B. FALSE: Labial herpes can occur but is far less common.
C. FALSE: Genital HSV may occur and should always be concerning for sexual abuse.
E. FALSE: Eczema herpeticum requires the HSV virus to infect a patient simultaneously with underlying atopic
dermatitis, Darier disease, or pemphigus vulgaris.
12. Which of the following statements regarding neonatal herpes simplex virus infection is correct?
A. Women with a primary HSV infection always demonstrate signs of disease.
B. Viral shedding and infectivity occur for a longer period in women with recurrent HSV
infection.
C. Rates of maternal-fetal transmission are lower in women with recurrent HSV infection
compared to women with primary infection.
D. A majority of women with active infection at the time of delivery are symptomatic.
E. Cesarean section is only necessary if there are active lesions in a primary HSV infection.
ANSWER: C.
EXPLANATIONS:
C. TRUE: Rates of maternal-fetal transmission of herpes simplex virus (HSV) are estimated to be as low as 2% in
infants born to mothers with recurrent HSV infection. However, the transmission rate of HSV from mother to
neonate is as high as 50% in infants born to mothers in the midst of a primary herpes infection. It is hypothesized
that the presence of protective maternal antibodies to the HSV virus in recurrently infected women contribute to
the lower rates of transmission.
A and D. FALSE: A significant proportion of women undergoing a primary HSV infection do not have recognizable
systemic or local disease. It is up to the physician to determine the risk of maternal-fetal transmission if the status
of the pregnant mother is in question. Additionally, a majority of women with active infections at the time of
delivery are asymptomatic, requiring astute clinical exam skills to determine the presence or absence of HSV
lesions in the birth canal.
B. FALSE: Viral shedding takes place at a significantly diminished amount of time in women with recurrent HSV
infection. The limited time of viral shedding (2-5) days compared to women with primary HSV infection (who can
shed for up to 2-3 weeks) is a contributing factor to the lower rates of maternal-fetal transmission in infants born
to mothers with recurrent HSV. This is due to specific humoral and cellular immunity.
E. FALSE: Cesarean section is indicated in women who have active genital lesions or prodromal symptoms at the
time of delivery, regardless of HSV infection status (primary vs. recurrent).
13. A 46-year-old woman visiting from Japan reports to the emergency room with a fever, arthralgias, and
generalized erythematous macules and papules. On further questioning, she notes that she owns a pet rat that
she often handles. Which of the following is true of this condition?
A. The causative agent, Spirillium minus, is a gram-positive rod.
B. The causative organism cannot easily be cultured.
C. Household pets other than rodents do not carry the disease.
D. Penicillin or tetracyclines are the first line of treatment.
E. There is a long incubation period of 1-2- months associated with this condition.
6
ANSWER: D.
EXPLANATIONS:
D. TRUE: The condition in this clinical vignette is rat-bite fever, and the causative agent is Streptobacillus
moniliformis. First-line treatment of rat-bite fever includes penicillins or tetracyclines.
A. FALSE: S. minus is a gram-negative rod.
B. FALSE: Culture is the typical mode of diagnosis of rat-bite fever.
C. FALSE: Cats and dogs can carry the bacteria and spread it.
E. FALSE: The incubation period is usually several days to less than 2 weeks.
14. A 36-year-old woman presents with violaceous papules and hemorrhagic vesicles on the palms and soles. She
reports a fever, chills, and malaise over the last few days. On review of systems, she notes joint pain in her right
elbow. There are lesions similar to those on her palms/soles overlying this joint. Which of the following
antibiotics should be given to this patient?
A. IV ceftriaxtone
B. PO doxycycline
C. IV vancomycin
D. IV vancomycin and IV ciprofloxacin
E. PO amoxicillin/clavulanate
ANSWER: A.
EXPLANATIONS:
A. TRUE: This woman has gonococcemia caused by Neisseria gonorhoeae. Patients experience fevers, chills, and
malaise. They will also note joint pain. Typical violaceous/hemorrhagic lesions will appear on the palms, soles, and
overlying joints. The treatment of choice for gonococcemia is IV ceftriaxone.
15. The highest risk of fetal varicella syndrome in pregnant women would be associated with which of the following
scenarios?
A. 30-week pregnant woman who developed herpes zoster
B. 20-week pregnant woman who developed herpes zoster
C. 30-week pregnant woman who developed a varicella infection
D. 20-week pregnant woman who developed a varicella infection
E. These scenarios place the fetus at equal risk for neonatal varicella syndrome.
ANSWER: D.
EXPLANATIONS:
D. TRUE: The highest risk of fetal varicella syndrome secondary to maternal varicella infection occurs between the
13th and 20th week of gestation at a rate of 2%. While still at risk, the rate of fetal varicella drops dramatically
before and after this period to approximately 0.4% prior to 13 weeks. No studies have demonstrated a significant
risk to the fetus when born to mothers who develop herpes zoster during pregnancy.
7
16. Four days following intense erythema of the face, which was mistaken for a sunburn by his parents, an 8-year-
old child is brought to your clinic with the clinical presentation illustrated below. Aside from the onset of this
new rash, he is in no acute distress and denies any significant symptoms. His parents report that he was not
feeling well 2 weeks prior and may have had an upper respiratory tract infection. Which of the following lab
abnormalities will aid your diagnosis?
A. Elevated hemoglobin
B. Decreased reticulocyte count
C. Decreased leukocyte count
D. Elevated liver transaminases
E. Seropositive anti-nuclear antibody (ANA)
ANSWER: B.
EXPLANATIONS:
B. TRUE: Parvovirus B-19 can manifest with a lacy reticular eruption of the extremities several days following the
typical slapped cheek appearance and sometimes occurs prior to the resolution of the facial erythema. This
occasionally pruritic eruption typically begins as confluent faint pink to erythematous patches on the arms and
legs, followed by central clearing and the reticular pattern pictured. At this stage of presentation, when the
infectious and viremic stage has passed, several lab changes can be noted. Parvovirus has a significant tropism for
erythroid progenitor cells, most importantly erythrocytes (more specifically a surface antigen on erythrocytes
known as P-antigen or globoside). This affinity is the pathogenesis of erythrocyte destruction, which can be
measured by a decreased reticulocyte count and decreased hemoglobin values.
Healthy patients with a parvovirus B infection experience some degree of hemolysis, which has a low potential to
present clinically. Transient aplastic crises can occur in patients who have parvovirus B infection and concomitant
diseases that are characterized by increased red blood cell destruction or decreased red blood cell production (e.g.
iron deficiency anemia, spherocytosis, sickle-cell disease, and glucose-6-phosphate dehydrogenase deficiency).
Prior to the conclusion of the prodromal phase, which lasts 7-10 days and begins 3-4 days following inoculation,
the leukocyte count increases and peaks prior to and during the onset of the cutaneous signs of erythema
infectiosum.
A. FALSE: The pathogenesis of erythrocyte destruction is measured by a decreased reticulocyte count and
decreased hemoglobin values.
C. FALSE: A decreased leukocyte count takes place only during the initial infectious state and does not occur with
the presentation of the facial erythema, reticular extremity rash, or arthralgias associated with parvovirus B-19.
D and E. FALSE: Elevated liver transaminases and anti-nuclear antibody (ANA) are not features of parvovirus-B
infection. However, the facial erythema of erythema infectiosum can clinically mimic the malar rash of systemic
lupus erythematosus as well as phototoxic eruptions and should be excluded from the differential diagnosis.
8
ANSWER: B.
EXPLANATIONS:
B. TRUE: The pathogenesis of impetigo is best described as a bacterial production of an exfoliating toxin (ETA or
ETB) by phage group II of Staph aureus at localized sites of infection.
A. FALSE: In Staphylococcus scalded skin syndrome (SSSS), the same toxins are circulated systemically and lead to a
more diffuse response.
C and D. FALSE: Autoimmune attacks on desmoglein 1 and 3 are responsible for mucous membrane and cutaneous
pemphigus vulgaris, and autoimmune attacks on desmoglein 1 are responsible for pemphigus foliaceus.
E. FALSE: Inherited mutations in intercellular adhesion proteins such as keratins 5 and 14 are the underlying
mechanism for inherited blistering disorders such as epidermolysis bullosa simplex.
18. A 3-year-old male is referred to your clinic for an eczematous rash by a local pediatrician. Three weeks prior, he
had a mild upper respiratory tract infection but otherwise has no significant past medical history. The rash of 2
weeks appears to involve the right axilla, right lateral flank, and right thigh, but the mother reports worsening and
spread to the left side of his body. They were given a topical corticosteroid by the pediatrician for itching that has
been of minimal help. The next step in managing this patient consists of which of the following?
A. Biopsy of the skin lesion
B. Viral culture
C. Reassurance
D. Oral valacyclovir
E. Topical calcineurin inhibitors
ANSWER: C.
EXPLANATIONS:
C. TRUE: This patient presents with the typical findings associated with unilateral laterothoracic exanthem (ULTE).
This rash typically presents as a morbilliform or eczematous eruption confined to a single side of the body,
originating in the axillae, progressing downward, and occasionally extending to the arm, waist, and thigh. With
time, lesions can extend to the contralateral aspect of the body, but the predominance of skin involvement
remains on the initial side involved. It can be preceded by viral upper respiratory or gastrointestinal illnesses and
has an approximately 3-6-week presence before resolving on its own. Occasionally, there can be associated
pruritus, but topical corticosteroids are of minimal help.
A. FALSE: A biopsy of the skin lesion would not aid in the diagnosis, as the histological features are nonspecific.
B. FALSE: Although associated with viral infections, the cutaneous lesions themselves would yield no information
since they are not virally induced.
D. FALSE: Despite ULTE's similarities to herpes zoster, which also presents in a well-defined distribution
unilaterally, oral valacyclovir would not affect the outcomes of patients with ULTE.
Similarly, topical calcineurin inhibitors would not be necessary for the treatment of this patient since lesions and
symptoms tend to resolve in a timely manner. Supportive therapy and reassurance that the rash will resolve is
satisfactory management for ULTE.
9
19. A 45-year-old man who lives in rural Pennsylvania visits the clinic for malaise, lethargy, and a very high fever.
On physical exam, there is an ulcerative lesion with a raised border on the right dorsal hand. Right axillary
lymphadenopathy is noted. A diagnosis of Tularemia is suspected. Which of the following statements is true of
tularemia?
A. The treatment of choice is doxycycline.
B. Francisella tularensis, the causative organism, is a gram-negative coccobacillus.
C. Sheep typically harbor the organism.
D. Chocolate agar is needed to culture the organism.
E. A conjugate vaccine is available for use in high-risk groups.
ANSWER: B.
EXPLANATIONS:
B. TRUE: F. tularensis is a gram-negative coccobacillus.
A. FALSE: The treatment of choice is streptomycin.
C. FALSE: F. tularensis is usually harbored by rabbits or rodents.
D. FALSE: Buffered charcoal and yeast extract (BCYE) agar is required to culture the organism.
E. FALSE: A vaccine is available, but it is a live attenuated vaccine.
20. A 15-year-old immunocompetent male presents with a single papulonodular lesion on his knee 2 weeks after
swimming in a nonchlorinated freshwater pool. Which of the following is the most likely diagnosis?
A. Mycobacterium marinum
B. Mycobacterium genavese
C. Mycobacterium haemophilum
D. Mycobacterium ulcerans
E. Mycobacterium xenopi
ANSWER: A.
EXPLANATIONS:
A. TRUE: M. marinum is associated with "swimming pool granuloma" and "fish tank granuloma." These are often
misdiagnosed as fungal or parasitic infections due to the rarity of this diagnosis as well as the usual difficulty in
eliciting a history of aquarium or freshwater tank/pool exposure. Growth enhances with 25°C growth and has the
following biochemical profile: Niacin +, Nitrate -, Catalase -, Urease +, and Tween 80 +.
Incorrect Answers:
B. M. genavese commonly occurs as disseminated disease in HIV+ patients.
C. M. haemophilum is usually seen in immunocompromised individuals and has been observed in lymphadenitis in
otherwise healthy children.
D. M. ulcerans infections are endemic to certain areas of Africa, Malaysia, Australia, New Guinea, and Mexico.
They are also known as Bairnsdale ulcer or Buruli disease.
E. M. xenopi is typically contracted from water sources and is more common in Canada and Europe than in the
United States.
21. A 45-year-old male who recently moved to the United States from Africa presents with a lesion on his foot that
has been growing for several weeks. The lesion is a tender deep firm lesion that has recently "opened up." The
lesion is draining a grainy whitish-yellowish substance. Which of the following is an unlikely causative agent?
A. Actinomadura madurae
B. Noetestudina rosatii
C. Aspergillus nidulans
D. Acremonium strictum
E. Madurella mycetomatis
10
ANSWER: E.
EXPLANATIONS:
E. TRUE: The lesion described is a mycetoma (previously termed actinomycetoma). A mycetoma is a chronic cutaneous
infection that can be caused by fungi (eumycetoma) or Actinomycetales bacteria. After skin inoculation, a pyogenic
response results in formation of a painless nodule at the site of entry. As the nodule enlarges, a chronic inflammatory
response occurs, causing indolent development of indurated lesions with sinus tracts that exude granules composed
of clumped organisms. M. mycetomatis causes mycetoma, but the drainage is typically black.
A. FALSE: A. madurae typically does cause whitish-yellowish drainage and could be a causative organism in this case.
B. FALSE: N. rosatii typically does cause whitish-yellowish drainage and could be a causative organism in this case.
C. FALSE: A. nidulans typically does cause whitish-yellowish drainage and could be a causative organism in this case.
D. FALSE: A. strictum typically does cause whitish-yellowish drainage and could be a causative organism in this case.
22. In patients with acute measles, which of the following supplementations is recommended to prevent significant
morbidity and mortality?
A. Vitamin K
B. Selenium
C. Vitamin C
D. Vitamin B12
E. Vitamin A
ANSWER: E.
EXPLANATIONS:
E. TRUE: The World Health Organization recommends treatment with vitamin A for all children with acute measles.
In children older than 12 months, 200,000 IU are given for 2 consecutive days, children 6-11 months of age should
receive 100,000 IU for the same time course, and children under 6 months of age should receive 50,000 IU for 2
days. No specific antiviral treatment exists for measles, but several studies have reported low serum levels of
vitamin A with increased measles-related morbidity and mortality, so the above recommendations should be
considered.
23. A 38-year-old male presents with complaints of a spot on his penis that has been there for a few weeks. On
physical exam, there is a painless soft erosion on the dorsal penis. There is also bilateral fluctuant inguinal
lymphadenopathy with the presence of a groove sign. A clinical diagnosis of lymphogranuloma venereum is
made. Which of the following genital ulcerative diseases is incorrectly matched with an appropriate treatment?
A. Chancroid – ciprofloxacin
B. Primary syphilis – penicillin
C. Granuloma inguinale – ciprofloxacin
D. Lymphogranuloma venereum – penicillin
E. Genital herpes – valacyclovir
ANSWER: D.
EXPLANATIONS:
D. This answer is correct. Lymphogranuloma venerum caused by C. trachomatis (serovars L1, L2, and L3) is treated
with Doxycycline, not penicillin.
A. A chancroid caused by H. ducreyi is typically treated with azithromycin, ceftriaxone, ciprofloxacin, or erythromycin.
B. Primary syphilis caused by T. pallidum is treated with penicillin.
C. Granuloma Inguinale caused by Klebsiella granulomatis is treated with bactrim, doxycycline, erythromycin, or
ciprofloxacin.
E. Genital herpes simplex infection is treated with acyclovir or valacyclovir.
11
24. A 23-year-old female presented to the emergency department with her third episode (in the past 3 years) of
rapidly developing diffuse petechial eruption. She reported this episode began shortly after developing fevers,
headaches, and hypotension. Blood cultures grew gram-negative diplococci. She was not on any long-term
medications. Following treatment initiation, which diagnostic study would be most useful for the long-term
management of this patient?
A. Isolation of the specific serogroup of diplococci to evaluate for pharyngeal colonization
B. Lumbar puncture to rule out CSF involvement
C. Coagulation studies (i.e. PT/PTT/INR)
D. Serum immunoglobulin G levels
E. Serum complement levels
ANSWER: E.
EXPLANATIONS:
E. TRUE: The complement cascade is an important component of the innate immune system. As opposed to IgG
deficiency, C8 deficiency more commonly presents for the first time in adolescence and adulthood. C8 is part of the
terminal complement components (C5-C9) that form the membrane attack complex (MAC). Humans with a
deficiency in a terminal complement component have an increased susceptibility to systemically invasive infection
with bacteria of the genus Neisseria.
A. FALSE: This case describes a presentation of recurrent meningococcemia in a young patient with possible C8
deficiency. Although false positive blood cultures can occur, recurrent bouts of a purpuric rash with signs of sepsis
suggest a true meningococcemia. Pharyngeal colonization may be asymptomatic or a source of infection, but there
are no data suggesting that decolonization prevents infection in C8-deficient patients.
B. FALSE: Although meningitis is an important step to rule out, treatment was already initiated (thus increasing the
false negative rate) and would not contribute to the long-term management of this patient.
C. FALSE: Coagulation disorders would not explain much of the symptomatology for this patient.
D. FALSE: Although immunoglobulins levels are an essential analysis for any immunodeficiency workup, recurrent
meningococcal infection should alert the practitioner to check complement levels. IgG deficiency affects adults and
children, but it most commonly presents in early childhood.
25. A 23-year-old man reports with complaints of malodorous feet and lesions on bilateral soles. The lesions are pits
that are 1 mm to 2 mm in diameter along with confluence forming thin surface erosions. Which of the following
statements concerning this condition is incorrect?
A. It is caused by bacteria of genus Kytococcus, Corynebacterium, or Streptomyces.
B. The lesions are due to proteases that break down the stratum corneum.
C. Oral antibiotic therapy is rarely needed to resolve the condition.
D. A moist environment promotes the development of this condition.
E. It can occur on the palms, but palmar lesions are more likely to present as erythematous
pinpoint papules.
ANSWER: E.
EXPLANATIONS:
E. TRUE: This is the incorrect statement. Palmar lesions often present as collarettes of scale. The condition in this
clinical vignette is pitted keratolysis.
A. FALSE: These bacteria are likely causes of pitted keratolysis.
B. FALSE: Proteases cause the pitted lesions.
C. FALSE: Topical therapies are the mainstay of treatment.
D. FALSE: Moist environments promote the development of pitted keratolysis.
12
26. A patient presents with vaginal lesions resembling the oral lesions illustrated below. What is the first-line
therapy for this condition?
A. Methotrexate
B. Mycophenolate mofetil
C. Oral prednisone
D. Superpotent class I corticosteroids
E. Topical vitamin D analogues
ANSWER: D.
EXPLANATIONS:
D. TRUE: Superpotent class I corticosteroids and calcineurin inhibitors are first-line therapies in the management of
oral and vulvovaginal LP. Clinical improvement was demonstrated rapidly with the use of tacrolimus 0.1%
ointment. While highly accepted as a mainstay of treatment, high-potency topical corticosteroids have no current
long-term comprehensive systematic reviews regarding efficacy and outcomes.
A, B, and C. FALSE: When topical corticosteroids and/or calcineurin inhibitor fails, immunosuppressive drugs should
be considered.
E. FALSE: Topical vitamin D analogues are not part of the management of oral LP or vulvovaginal LP.
27. Which of the following diseases has been associated with polyomavirus?
A. Erythema infectiosum
B. Roseola infantum
C. Mollsucum contagiosum
D. Merkel cell carcinoma
E. Milker’s nodule
ANSWER: D.
EXPLANATIONS:
D. TRUE: Merkel cell carcinoma was originally named trabecular carcinoma based on its lattice-like and infiltrative
growth pattern. Roughly 80% of Merkel cell carcinomas are positive for polyomavirus.
A. FALSE: Erythema infectiosum (aka “slapped cheek” disease) is caused by parvovirus B19 infection.
B. FALSE: Roseola infantum (aka exanthema subitum) is caused by HHV-6 and 7 infection.
C. FALSE: Molluscum contagiosum is caused by poxvirus infection.
E. FALSE: Milker’s nodules are caused by paravaccinia/parapox infection.
28. A 2-year-old patient who has just moved to the United States presents for his first checkup. He was delivered at
home, and the mother noted that the newborn was "quite small" and had some "snuffles." The possibility of
congenital syphilis is considered. Which of the following is unlikely to occur in this patient?
A. Presence of mulberry molars
B. Saber shins
C. Trigeminal nerve palsy
D. Gummas of the palate
E. Peg-shaped teeth
13
ANSWER: C.
EXPLANATIONS:
C. TRUE: Congenital syphilis can be divided into early and late stages. The early stage occurs prior to 2 years of age
and consists of low birth weight, saw-toothing of bone metaphyses, rhinitis (snuffles), epitrochlear
lymphadenopathy, and neurosyphilis. The late stage after 2 years of age may present with mulberry molars, peg
teeth, dactylitis, high arched palate, saber shins, saddle nose, enlargement of medial clavicle, retinitis, gummas of
palate and nose, and eighth nerve deafness, and keratitis. Eighth nerve deafness, keratitis, and peg teeth make up
Hutchinson’s triad. Trigeminal nerve palsy is not a typical manifestation of late congenital syphilis.
29. A patient with multibacillary leprosy is receiving dapsone, clofazimine, and rifampin. Which of the following
statements is true regarding the mechanism of action of these medications?
A. Dapsone is bacteriostatic because of its inhibitory effects on dihydrofolate reductase.
B. Clofazimine is bactericidal by directly inhibiting bacterial DNA polymerase.
C. Rifampin is bacteriostatic by inhibiting RNA synthesis by blocking DNA-dependent RNA
polymerase.
D. Rifampin is bactericidal by inhibiting RNA synthesis via blocking DNA-dependent RNA
polymerase.
ANSWER: D.
EXPLANATIONS:
C/D. TRUE: Rifampin is bactericidal by inhibiting RNA synthesis via blocking DNA-dependent RNA polymerase.
A . FALSE: Dapsone inhibits bacterial synthesis of dihydrofolic acid via competition with para-aminobenzoate for
the active site of dihydropteroate synthetase. Dapsone is both bacteriostatic and weakly bactericidal against M.
leprae. Neither of the listed mechanisms is the cause of these effects.
B. FALSE: As a substance with both anti-leprosy and anti-inflammatory activity, clofazimine is weakly bactericidal
against M. leprae by binding to the guanine bases of bacterial DNA, not DNA polymerase directly.
30. A 10-year-old male with a history of mild to moderate eczema has numerous asymptomatic lesions (pictured
below). Which of the following is correct regarding the histology shown in this otherwise healthy patient?
14
ANSWER: E.
EXPLANATIONS:
E. TRUE: The figure displays the histologic appearance of molluscum contagiosum. These lesions may be treated by
cantharidin, curettage, and retinoids.
A. FALSE: Molluscum contagiosum demonstrates virions that have proliferated in the cytoplasm of epithelial cells,
resulting in characteristic intracytoplasmic bodies (Henderson-Patterson bodies).
B. FALSE: This is an infectious process and is spread by direct inoculation. It is unlikely to spread hematogenously.
C. FALSE: Molluscum contagiosum is caused by the infection of superficial portions of the follicular units by a DNA
poxvirus, not human papillomavirus. The inclusions are cytoplasmic.
D. FALSE: Besides watchful waiting, cryosurgery, curette scraping, and cantharidin are all options preferred over
excision, as these lesions eventually resolve spontaneously.
31. A 14-year-old girl presents with a 3-year history of yellow-crusted plaques and extensive hair loss. There is
peripheral scutula formation as well. KOH examination of the hairs reveals infected hair filled with hyphae
producing bubbles of gas. What is the most likely cause of this condition?
A. Trichophyton rubrum
B. Trichophyton tonsurans
C. Microsporum canis
D. Trichophyton schoenleinii
E. Aspergillus sp.
ANSWER: D.
EXPLANATIONS:
D. TRUE: Scutula formation, chronicity, and extensive hair loss are clinically consistent with tinea favosa (favus), a
severe form of tinea capitis caused by T. schoenleinii. In tinea favosa, upon exposure to KOH, there is bubbling of
KOH through the air spaces between hyphae elements. This severe form of hair infection is not typical of other
causes of tinea capitis such as T. tonsurans and M. canis. Rarely, favus is caused by Trichophyton violaceum,
Trichophyton mentagrophytes var quinckeanum, or Microsporum gypseum. None of the other options have been
reported to cause favus.
32. A 4-year-old child presents to the clinic with a recent viral type illness. In addition to his typical viral
symptoms, he has erythematous papules and vesicles with a surrounding darker erythematous border on
palms, soles, and perioral area. Which of the following is true of this infection?
A. It is caused by Coxsackievirus A16 and Enterovirus B72.
B. Oral-fecal transmission is the sole mechanism of spread.
C. The fever and skin rash appear at the same time.
D. Patients can expect to be affected for 3-4 weeks before resolution.
E. It has an incubation period of several days to 1 week.
ANSWER: E.
EXPLANATIONS:
E. TRUE: The clinical vignette describes a child with hand-foot-mouth disease. The typical incubation period is a
few days to about 1 week.
A. FALSE: Hand-foot-mouth disease is caused by Coxsackievirus A16 and Enterovirus A 71.
B. FALSE: Oral-oral and oral-fecal modes of transmission both occur.
C. FALSE: The rash usually follows the fever by 1-2 days.
D. FALSE: The course of illness typically lasts for about 1 week.
15
33. A 17-year-old male presents with yellowish crusts on his bilateral axillary hairs. He relates that this symptom
started about a month ago, soon after he started frequenting the gym. He has no other symptoms or skin
complaints. What is the most likely causative organism?
A. Tinea verrucosum
B. Corynebacterium minutissimum
C. Microsporum gypseum
D. Corynebacterium tenuis
E. Tinea schoenleinii
ANSWER: D.
EXPLANATIONS:
D. TRUE: This is a young patient with Trichoymycosis Axillaris. The name of this condition is a misnomer in that the
causative agent is a bacterium (Corynebacterium tenuis), not a fungus.
A. FALSE: Tinea verrucosum is a fungus that causes tinea capitis.
B. FALSE: Corynebacterium minutissimum typically causes erythrasma.
C. FALSE: Microsporum gypseum is a ground-dwelling dermatophyte.
E. FALSE: Tinea schoenleinii is a fungus that causes tinea capitis (flavus).
34. A 48-year-old marine park dolphin trainer presents with a several-year history of slowly growing bumps (see
photo). The patient stated that this condition initially started as a small bump that did not resolve with topical
antifungal use. The lesions are occasionally pruritic but otherwise asymptomatic. Which of the following is
correct regarding this condition?
ANSWER: A.
EXPLANATIONS:
A. TRUE: The figure displays the clinical appearance of lobomycosis (keloidal blastomycosis). These lesions are
quite insidious, thus presenting many years following inoculation. There has been only 1 reported case in the
United States, with most reports occurring in the Amazon basin of Brazil.
B. FALSE: Up to 25% of patients can develop lymph node involvement.
C. FALSE: Histologic findings are characteristic for chronic granulomatous inflammation with surrounding keloidal
development. Fungi are typically present in tissue and arranged as chains of yeast-like cells that are approximately
10 μm in diameter, with thick walls and interconnecting tubular structures which later disappear.
16
D. FALSE: Surgical excision is the definitive therapy in these patients, as only a few case reports have reported
successful treatment with oral antimicrobial therapy (clofazimine).
E. FALSE: Besides the skin lesions and occasional lymphadenopathy, patients lack systemic findings.
35. A 2-year-old male presents to the emergency room with 2-3 days of fevers, chills, and diarrhea. On
examination, you note painful oral erosions and tender red papules on the palms and soles. The mother states
that the patient has not been eating well but was able to take some acetaminophen for the fevers. Which of the
following describes the cause of this condition?
A. RNA retrovirus
B. Negative-sense RNA virus
C. Positive-sense RNA virus
D. Double-stranded DNA virus
E. Single-stranded DNA virus
ANSWER: C.
EXPLANATIONS:
C. TRUE: This describes a case of hand-foot-and-mouth disease. Coxsackievirus A16, a picornavirus, is the most
common cause of hand, foot, and mouth disease in the United States. Picornaviradae are single-stranded, positive-
sense RNA viruses. Enterovirus 71, also a picornavirus, has been implicated in outbreaks as well.
36. A 47-year-old female developed an acneiform facial eruption at the time of elective ambulatory surgery when
she was treated with preoperative and postoperative antibiotics. Occasionally, the patient will develop painful
nodules as well. Topical tretinoin cream and doxycycline have not been effective. Culture of a pustule reveals
the growth of Serratia marcescens. What is the next step in the management of this patient?
ANSWER: E.
EXPLANATIONS:
E. TRUE: This a classic case of gram-negative folliculitis. Although this condition can arise de novo, typically it
presents in patients with extensive antibiotics history and is recalcitrant to traditional acne therapy. Prior to
initiating isotretinoin, baseline lab work, pregnancy testing, informed consent, and registration to the iPledge
program are required. De novo gram-negative folliculitis due to Pseudomonas may occur, but it is often self-
limited, resolving in 7 to 10 days with good skin hygiene. Avoidance of contaminated water and proper
chlorination of pools and hot tubs is preventative. Oral ciprofloxacin (250 to 750 mg twice daily) can be used for
severe cases, but oral isotretinoin is the treatment of choice for recalcitrant disease.
A. FALSE: Serratia is a facultative gram-negative organism found in the GI tract of children and the urinary or
respiratory tracts of adults. This is not normal skin flora.
B. FALSE: Minocycline has a broad spectrum of activity including both gram-positive and gram-negative organisms,
however, the poor efficacy of doxycycline suggests the organism is likely resistant to tetracyclines. Better antibiotic
choices for gram-negative folliculitis include extended-spectrum penicillins and trimethoprim-sulfamethoxazole.
17
C. FALSE: Although benzoyl peroxide does have antibacterial properties, it would not be the solution for a
recalcitrant gram-negative folliculitis.
D. FALSE: Metronidazole gel is rarely helpful in gram-negative folliculitis.
37. A patient has recurrent furuncles that have evolved into larger abscesses and are no longer responding to
topical erythromycin. You suspect he may have methicillin-resistant Staphylococcus aureus (MRSA), and
cultures return confirming this suspicious. Which of the following treatments is inappropriate?
A. Trimethoprim-sulfamethoxazole (TMP-SMX)
B. Minocycline
C. Clindamycin
D. Mupirocin
E. Linezolid
ANSWER: D.
EXPLANATIONS:
D. TRUE: Mupirocin is a topical antibiotic and would not be appropriate in the treatment of any MRSA-positive
infection except for a superficial infection. MRSA infections are divided into minor and serious infections. Minor
infections can include furuncles, abscesses, purulent cellulitis, and infections that have failed to demonstrate a
response to first-line anti-staphylococcal beta-lactams such as penicillins or cephalosporins. First-line therapy for
MRSA-positive minor infections includes TMP-SMX, minocycline, doxycycline, or clindamycin. If MRSA is further
resistant to these medications, linezolid is often second-line. Serious MRSA infections, in which patients are
systemically ill and require hospitalization, require immediate treatment with IV vancomycin; linezolid or
daptomycin are used as second-line antibiotics. It is important to inquire about local resistance patterns prior to
starting non-empiric treatment of MRSA since patterns change from community to community.
38. A 58-year-old veterinarian reports to the ED for a single persistent papular lesion on his dorsal index finger. It
started as a red bump that recently became whitish and is draining fluid. He has no other systemic symptoms.
On further questioning, the veterinarian relates that he treats both household pets and larger farm animals.
Which of the following is true concerning this patient’s condition?
A. The causative agent is a poxvirus.
B. The lesion described typically progresses through 3 phases.
C. The virus can live for up to 2 weeks without a natural host.
D. There is a human vaccine against the offending agent.
E. The lesion typically lasts for 2 weeks.
ANSWER: A.
EXPLANATIONS:
A. TRUE: The virus that causes orf (contagious pustular dermatitis) is a poxvirus.
B. FALSE: Orf lesions progress through 6 phases: papular, targetoid, acute, regenerative, papillomatous, and
regressive.
C. FALSE: The orf virus can live in soil for up to 6 months.
D. FALSE: There is a vaccine for livestock, but it can cause disease in humans.
E. FALSE: The orf lesion typically persists for 7-10 weeks.
18
39. An 8-year-old male immigrant from South America presents with several warty growths on his right arm. His
mother relates that the growths started as a painless papule that turned into a large warty nodule. Several
smaller identical nodules have appeared around it. A diagnosis of yaws is considered. Which of the following
pairs of Treponeme and causative disease is incorrect?
A. T. pallidum denticola - periodontitis
B. T. pallidum pertenue - yaws
C. T. pallidum pallidum - syphilis
D. T. pallidum carateum - pinta
E. All of these pairings are correct.
40. Which of the following medical conditions may predispose patients to Staph aureus infections of the entire hair
follicle unit?
A. Chronic granulomatous disease
B. Hyperimmunoglobulin E syndrome
C. Diabetes mellitus
D. Obesity
E. All of the above
ANSWER: E.
EXPLANATIONS:
E. TRUE: When Staphylococcus aureus involves the entirety of a hair-follicle unit, leading to collections of pus that
are walled off from the surrounding tissues, it is known as furunculosis. A singly involved and infected hair follicle
unit is known as a furuncle. Carbuncles are collections of numerous furuncles and extend deeper into subcutaneous
tissue. Many immunodeficiencies that prevent the necessary steps to identify (e.g. hyper IgE syndrome) and then
eradicate (e.g. chronic granulomatous disease) gram-positive Staphylococcus aureus, predispose individuals to
recurrent hair-follicle abscesses and inflammation of surrounding tissue. Additionally, body habitus (obesity) as
well as poor glycemic control (diabetes mellitus) provide an ample environment for reproduction and continued
infection of hair follicles by Staph aureus, leading to cutaneous furuncles, carbuncles, and abscesses.
ANSWER: D.
EXPLANATIONS:
D. TRUE: Penicillins are the first-line treatment as long as the patient is not allergic. Treatment may take months to
be successful.
A. FALSE: Actinomycosis is most often caused by Actinomyces israelii.
B. FALSE: The etiology of actinomycosis is typically endogenous bacteria such as oral flora.
C. FALSE: The peri-mandibular and cervical areas are most commonly affected.
19
42. Which of the following pairs of human herpesvirus (HHV) and related conditions is incorrectly matched?
A. HHV-2 – oral herpes and genital herpes
B. HHV-4 – infectious mononucleosis and Castleman’s disease
C. HHV-6 – exanthem subitum
D. HHV-3 – chicken pox and shingles
E. HHV-8 – Kaposi sarcoma
ANSWER: B.
EXPLANATIONS:
B. TRUE: HHV-4 causes infectious mononucleosis and lymphoma, but HHV-8 causes Castleman’s disease.
A. FALSE: HHV-2 causes oral and genital herpes (as does HHV-1).
C. FALSE: HHV-6 causes exanthem subitum.
D. FALSE: HHV-3 causes chicken pox and shingles.
E. FALSE: HHV-8 causes Kaposi sarcoma (as well as Castleman’s disease).
43. A 1-year-old infant presents to the clinic with a high fever that started 5 days ago. Over the last 24 hours, the
infant has developed erythematous pink macules and papules with surrounding light rings or halos. The
infant’s mother, who has been taking the infant’s temperature several times a day, states that the fever has
improved since the rash appeared. Which of the following statements regarding this infant’s condition is true?
A. The fever occurs gradually while the rash appears suddenly.
B. The rash usually forms first on the limbs and then spreads to the trunk.
C. Reactivation can occur in an adult who was infected as a child.
D. It is most likely caused by HHV- 8.
E. Cidofovir is indicated as the treatment of choice.
ANSWER: C.
EXPLANATIONS:
C. TRUE: The clinical vignette describes an infant with exanthem subitum. Adults may experience reactivation
manifesting as a less severe illness.
A. FALSE: The fever and rash typically develop suddenly in exanthem subitum.
B. FALSE: The rash in exanthem subitum typically occurs on the trunk first, then spreads to the limbs.
D. FALSE: HHV-6 and HHV-7 are the causes of exanthem subitum.
E. FALSE: Some success has been noted with the use of cidofovir, but it is not a recommended treatment at this time.
44. Which of the following Rickettsial diseases is incorrectly matched with its causative organism?
A. Rocky Mountain spotted fever - Rickettsia rickettsii
B. Epidemic typhus - Rickettsia prowazekii
C. Scrub typhus - Rickettsia akari
D. Q fever - Coxiella burnetii
E. Ehrlichiosis - Ehrlichia chaffeensis
ANSWER: C.
EXPLANATIONS:
C. This answer is correct. Scrub typhus is caused by Orientia tsutsugamushi. R. akari causes Rickettsialpox.
A. Rocky Mountain spotted fever is caused by R. rickettsii.
B. Epidemic typhus is caused by R. prowazekii. Endemic typhus is caused by R. typhi.
D. Q fever is caused by C. burnetii.
E. Ehrlichiosis is caused by E. chaffensis.
20
45. A 32-year-old woman who recently immigrated from India presents with a lesion on her dorsal right hand.
According to the patient, the lesion began as an odd-looking red bump. She relates that the bump grew a bit
and became fluid-filled. Over the last few weeks, the lesion no longer contains fluid but has formed a black
crust. Which of the following is true of the disease associated with this lesion?
A. It is likely caused by a gram-negative spore-forming rod.
B. The causative agent produces 2 types of exotoxins.
C. Azithromycin is a first-line treatment.
D. There are 2 clinical forms of this disease: inhalational and cutaneous.
E. None of the above
ANSWER: B.
EXPLANATIONS:
B. TRUE: The likely etiology of this lesion is anthrax, caused by Bacillus anthracis. B. anthracis produces both
edema toxin and lethal toxin.
A. FALSE: The causative organism of the lesion is a gram-positive spore-forming rod.
C. FALSE: Doxycycline or ciprofloxacin are first-line treatments for anthrax.
D. FALSE: There are 3 clinical forms of anthrax: inhalational, gastrointestinal, and cutaneous.
46. Match the mechanism of action with the following antifungal agents.
ANSWER: A.
EXPLANATIONS:
A. TRUE: Flucytosine interferes with DNA and RNA synthesis selectively in fungi. It is synergistic with amphotericin.
It may cause systemic toxicity in the host due to DNA and RNA effects. Ketoconazole causes a blockade of fungal
P450 enzymes and also interferes with ergosterol synthesis. It is poorly selective as compared to itraconazole. It
also interferes with mammalian P450 function. Caspofungin blocks the β glucan synthase and prevents synthesis of
the fungal cell wall. Terbinafine inhibits epoxidation of squalene in fungi; accumulation of increased levels of
squalene is toxic to fungi.
47. Which of the following infections with Staphylococcus aureus does not warrant a bacterial culture of cutaneous
lesions?
A. Bullous impetigo
B. Abscess
C. Non-bullous impetigo
D. Furunculosis
E. Staph scalded skin syndrome
21
ANSWER: E.
EXPLANATIONS:
E. TRUE: All of the conditions listed involve cutaneous manifestations that can be cultured and sent to the lab for
confirmation of infection and necessary information regarding bacterial sensitivity to antibiotics. This includes ruling
out infection with methicillin-resistant Staphylococcus aureus (MRSA). The pathogenesis underlying the cutaneous
features of Staphylococcus scalded skin syndrome is due to the systemic circulation of the exfoliating toxins A and B,
resulting in findings that would not grow bacteria with a culture from the involved skin. In children, the infectious
focus typically resides in the nasopharynx or conjunctiva, and culture from these areas may be more high-yield.
A and C. FALSE: Both bullous and non-bullous impetigo are due to the localized production of exfoliating toxin A and B
without the systemic spread of these toxins.
B and D. FALSE: Abscess and furunculosis are secondary to bacterial invasion into the skin and hair follicle respectively
with a localized surrounding inflammatory response.
48. A 42 year old fisherman presents to clinic with a well demarcated erythematous and violaceous lesion with
central clearing and elevated margins located on the dorsal hand. What is the first line treatment for this
condition?
A. Penicillin
B. Doxycycline
C. Vancomycin
D. Erythromycin
E. Trimethoprim/Sulfamethoxazole
49. A 9-year-old girl presents to the pediatrician with a recent flu-like illness. Just after the flu-like illness started, a
disseminated rash of erythematous papules began. Some of the papules began to turn into vesicles as more
papules arose. Eventually, some of the vesicles began to crust over, but many are currently either erythematous
papules or vesicles. Which of the following is true of this illness?
A. Infection of a pregnant woman from 5 days before to 2 days after delivery can cause
congenital varicella syndrome.
B. An individual lesion will evolve from macule to papule to vesicle to crust over 5 days.
C. The highest risk of congenital varicella syndrome occurs within the first 20 weeks of
pregnancy.
D. The varicella vaccine consists of heat-killed virus.
E. Disseminated herpes zoster is defined as 5 lesions outside an affected dermatome.
22
ANSWER: C.
EXPLANATIONS:
C. TRUE: This patient has varicella-zoster virus (VZV) infection. The highest risk of congenital varicella syndrome
occurs within the first 20 weeks of pregnancy. The syndrome consists of hypoplastic limbs, ocular abnormalities,
and CNS abnormalities.
A. FALSE: Infection during this time can cause neonatal varicella, not congenital varicella syndrome.
B. FALSE: The evolution of a varicella lesion typically occurs over 1-2 days.
D. FALSE: The varicella vaccine consists of a weakened virus.
E. FALSE: Twenty or more lesions outside the affected dermatome is diagnostic of disseminated herpes zoster.
50. A 22-year-old female patient presents to the clinic for a new onset rash that appeared fairly suddenly and is
very pruritic. On physical exam, there are numerous erythematous papules scattered on the buttocks and
bilateral breasts. Other pertinent history includes a recent trip to the beaches of the Florida coast, where she
spent a lot of time in the water. She is wondering if something from the trip caused this odd rash and asks if she
got bed bugs from the hotel room where she stayed. What is the correct response to her question?
A. "No, this is not bed bugs. It is something called swimmer's itch."
B. "Yes, this is likely bed bugs. Be sure to wash all your clothing and bed linens."
C. "No, this is not bed bugs. However, it is caused by an organism from the family
Schistosomatidae."
D. "Yes, this is likely bed bugs. I will prescribe you an oral anti-itch medication and a topical
steroid. You should also call the hotel to alert them."
E. "No, this is not bed bugs. I will prescribe you an oral anti-itch medication and a topical
steroid."
ANSWER: E.
EXPLANATIONS:
E. TRUE: The patient in this vignette has seabather's eruption, which is caused by the larvae of Edwardsiella lineata
and Linuche unguiculata. While in saltwater, swimmers will encounter the larvae, which become trapped under
swimming garments. The larvae have nematocysts that are automatically discharged when dried or when they
encounter fresh water. This causes a rash in the typical "bathing suit" distribution. Swimmer's itch is typically caused
by the freshwater forms of Schistomomatidae flatworms (saltwater forms do exist, however). Lesions appear on
uncovered skin. There is no special treatment for seabather's eruption or swimmer's itch other than symptomatic
relief.
51. A 13-year-old male is brought to your clinic following the development of erythema and edema of the palms and
soles. On physical exam, you note significant petechiae and purpura on the palms and soles. The patient is otherwise
healthy and reports only a mild fever. Which of the following is correct regarding this patient’s presentation?
A. These findings are consistent with infection by a gram-positive bacterium.
B. These findings are consistent with infection by an RNA virus.
C. These findings are consistent with infection by a retrovirus.
D. These findings are consistent with infection by a double-stranded DNA virus.
E. These findings are consistent with infection by a single-stranded DNA virus.
ANSWER: E.
EXPLANATIONS:
E. TRUE: The vignette describes the clinical findings consistent with papular purpuric gloves and socks syndrome
(PPGSS). This less common manifestation of parvovirus B19 infection differs from that of erythema infectiosum in
23
that it has a broader range of age involvement, typically affecting young adults, and can also be characterized by
an enanthem consisting of erosions and petechiae of the palate, pharynx, and tongue. Parvovirus B19 is a single-
stranded DNA virus.
52. A 36-year-old African immigrant visits the ED with unilateral swelling in his right leg. He is a recent immigrant and
is from an area of Africa where filariasis is common. This is the suspected diagnosis, and the appropriate workup is
initiated. In general, which of the following mosquito species is not a likely vector for lymphatic filariasis?
A. Aedes
B. Psorophora
C. Mansonia
D. Culex
E. Anopheles
ANSWER: B.
EXPLANATIONS:
B. TRUE: Lymphatic filariasis is a disease marked by lymphatic block due to parasitic organisms that causes
lymphedema and elephantiasis. The typical causative organisms are Brugia malayi, Brugia timori, and Wuchereria
bancrofti. Mosquitos are the primary vectors, namely Aedes, Mansonia, Culex, and Anopheles types. Psorophora
does not typically transmit filariasis. Other types of filariasis include subcutaneous and serous cavity filariasis.
53. A 27-year-old male patient presents to the clinic with a painful right ear. The ear is red and swollen. According
to the patient, he first experienced a dull pain in his ear with subsequent development of redness and warmth.
Which of the following is incorrect regarding the likely cause of this patient’s condition?
A. The ideal treatment is with a fluoroquinolone.
B. The most common cause is polymicrobial.
C. The infection is often precipitated by trauma.
D. Surgery may be necessary in severe cases.
E. Herpes zoster can precipitate the condition.
ANSWER: B.
EXPLANATIONS:
B. TRUE: This answer is incorrect. The most common cause is Pseudomonas aeruginosa.
A. FALSE: P. aeurginosa, the most common cause of infectious perichondritis, is sensitive to fluoroquinolones.
C. FALSE: Trauma is often a precipitating factor of infectious perichondritis.
D. FALSE: If infection causes necrosis of soft tissue and cartilage, surgical intervention may be required.
E. FALSE: Herpes zoster lesions can serve as a portal of entry for the bacteria.
54. A 43-year-old Mexican-American immigrant presents with anesthetic lesions as shown in the image below. A
diagnosis of Hansen’s disease (Leprosy) is suspected. Which of the following Ridley-Jopling classification
category is matched with incorrect characteristics?
24
ANSWER: A.
EXPLANATIONS:
A. TRUE: This is not correctly matched. Erythema nodosum leprosum is a type II reaction seen in patients with a high
bacterial load such as those with lepromatous or borderline lepromatous disease, not tuberculoid leprosy. Erythema
nodosum leprosum presents with painful ulcerative subcutaneous nodules on the extensor surfaces. It is thought to be
secondary to immune complex formation from M. leprae antibodies and antigens. Treatment is with thalidomide.
B. FALSE: BT lesions (and TT lesions) typically lack sensation and are anhidrotic.
C. FALSE: Borderline leprosy is the most common form.
D. FALSE: BL and LL forms of leprosy tend toward a multibacillary classification with a TH2 cytokine response versus
the TH1 response characteristic of BT and TT forms.
E. FALSE: Lepromatous lesions are generalized and symmetrical.
55. A company has developed a simple but effective method for quickly identifying dermatophytes in
onychomycosis. Although this method identifies dermatophytes, which of these organisms is most likely to test
negative but still cause onychomycosis?
A. Trichophyton rubrum
B. Exophiala werneckii
C. Fusarium sp.
D. Epidermophyton floccosum
E. Microsporum sp.
ANSWER: C.
EXPLANATIONS:
C. TRUE: Fusarium sp. molds are the most common cause of non-dermatophyte onychomycosis and would produce
a negative test.
A. FALSE: Although the majority of cases of onychomycosis are dermatophyte-related, several non-dermatophyte
molds can cause onychomycosis. Thus, a method of detecting dermatophytes would likely produce several false
negatives. T. rubrum, a dermatophyte, would not produce a negative test.
B. FALSE: Although E. wernickii is not a dermatophyte, it is a rare cause of onychomycosis and is more likely to
cause tinea nigra.
D. FALSE: E. floccosum is a dermatophyte and would not produce a negative test.
E. FALSE: Microsporum sp. are dermatophytes.
56. A 36-year-old pet store owner presents with a skin lesion on her dorsal hand. She says the lesion started as a red
bump but formed into a nodule. A rough area is noted presently. The patient also recalls cutting her hand
recently while cleaning the pet store's fish tanks. On physical exam, you notice an erythematous verrucous
indurated plaque on the dorsal hand, along with a small erythematous indurated patch on her forearm. Which
of the following antibiotics is indicated to treat this condition?
A. Vancomycin
B. Streptomycin
C. Amoxicillin
D. Minocycline
E. Ciprofloxacin
25
ANSWER: D.
EXPLANATIONS:
D. TRUE: Mycobacterium marinum infection is usually acquired through a pre-existing skin lesion followed by
inoculation from an aquarium or swimming pool harboring the bacteria. The infection causes a local lesion and may
demonstrate sporothrichoid spread. The indicated antibiotic treatment for M. marinum infection is minocycline.
57. You attend a morbidity and mortality conference where the discussion is focused on a patient who succumbed
to a neurodegenerative disorder following notable personality changes, prolonged coma, and eventual death.
Her past medical history was relatively uneventful, but her parents were not supportive of standard
vaccinations. The most likely cause of death can be attributed to which of the following infections?
A. Rubella
B. Rubeola
C. Parvovirus B19
D. Enterovirus
E. Varicella-zoster virus
ANSWER: B.
EXPLANATIONS:
B. TRUE: This patient succumbed to complications of non-vaccination to rubeola, also known as measles. This
syndrome is known as subacute sclerosing panencephalitis. It is a delayed neurodegenerative disease that presents
years after measles infection. Symptoms consist of personality changes, coma, and eventually death. Routine
vaccination against the measles virus is recommended to avoid these complications. The other options are not
associated with subacute sclerosing panencephalitis.
58. A 6-year-old male is brought to your clinic by his mother with concerns about his nails falling off. She mentions
that he just started attending daycare in the late spring, and within a few weeks developed a rash involving the
trunk, arms, legs, hands, and feet. There were no significant complications following the rash, and his daily
activities were not interrupted. Three months later, she noted that most of his fingernails and toenails have
begun to shed. Which of the following is the most likely cause of his onychomadesis?
A. Picornavirus
B. Paramyxoviridae
C. Herpesvirus
D. Togavirus
E. Poxvirus
ANSWER: A.
EXPLANATIONS:
A. TRUE: The most frequently encountered cause of onychomadesis (proximal shedding of the nails) is hand-foot-
mouth disease caused by Coxsackie viruses. Enteroviruses are a subgroup in the family of Picornaviridae that causes
a wide range of illnesses and exanthems. The most commonly encountered enteroviral manifestation is hand-foot-
mouth disease. It is characterized by a vesicular eruption on the hands and feet as well as erosive stomatitis. Not
every enterovirus infection manifests with the specific findings that preclude a diagnosis of hand-foot-mouth
disease, but symptoms including cutaneous findings are generally the same. There are cases of hand-foot-mouth
disease that present with no oral findings and cutaneous involvement in atypical areas such as the buttocks.
B. FALSE: Parmyxoviridae include parainfluenza virus and the viral cause of measles/rubeola.
C. FALSE: Herpesviridae include HSV 1 and 2, the Varicella-Zoster Virus, Cytomegalovirus and others.
D. FALSE: Togaviridae include Rubella and the viruses causing both Eastern and Western Equine encephalitis.
E. FALSE: Poxviridae include viruses responsible for cowpox, monkeypox, pseudocowpox, and smallpox.
26
59. A 40-year-old male patient with a history of chronic alcohol use disorder and HIV infection presents to the
emergency room for a several-month history of a nonhealing skin ulceration on the lower extremity. The
ulceration has a surrounding draining sinus with purulent material. A biopsy of the ulceration is shown below.
What is the most likely diagnosis?
A. Staphylococcus aureus infection
B. Human herpesvirus 8 infection
C. Squamous cell carcinoma
D. Sporotrichosis
E. Atypical mycobacterial infection
ANSWER: A.
EXPLANATIONS:
A. TRUE: The clinical vignette and histology are representative of botryomycosis. The most common causative
organism is Staphylococcus aureus. Botryomycosis presents with cutaneous “mycetoma-like” ulcerations with sinuses
and fistulae draining yellow granules. A biopsy shows basophilic sulfur granules with a surrounding hyaline periphery
due to host immunoglobulin response (Splendore-Hoeppli phenomenon).
B. FALSE: Human herpesvirus 8 is the cause of Kaposi sarcoma, an angioproliferative disorder seen in patients with
HIV. Pathology shows perivascular lymphocytes, angiogenesis, and spindled cells. A characteristic “promontory sign”
of a small vessel protruding into an abnormal vascular space can be seen in patch or plaque Kaposi sarcoma.
C. FALSE: A squamous cell carcinoma should be on the differential of any nonhealing wound in an immunosuppressed
patient. Pathology shows atypical keratinocytes invading into the dermis.
D. FALSE: Sporotrichosis or “rose gardener’s disease” presents with a cutaneous ulceration with lymphatic spread
following an inoculation and is more common among immunosuppressed patients. Histology shows
pseudoepitheliomatous hyperplasia and the presence of cigar-shaped spores.
E. FALSE: Atypical mycobacterial infections such as M. marium or M. chelonae are also on the differential diagnosis.
Histology presents with tuberculoid-like granulomas in addition to an intense inflammatory infiltrate. Acid-fast bacilli
can be observed after staining.
27
60. A neonate is born with generalized violaceous purpura. Congenital CMV is suspected. Which of the following is
incorrect concerning congenital CMV?
A. It is the most common infectious cause of deafness in the United States.
B. A woman infected with CMV 2 years prior to pregnancy carries little risk of delivering a baby
with congenital CMV.
C. It is the most common congenital infection.
D. It is the most common infectious cause of intellectual disability in the United States.
E. It occurs more often in pregnant women of lower socioeconomic status than in those of
higher socioeconomic status.
ANSWER: E.
EXPLANATIONS:
E. TRUE: This statement is false. Due to their increased likelihood of CMV seropositivity before pregnancy occurs,
women of lower socioeconomic status are less likely to deliver babies affected by congenital CMV.
A. FALSE: Congenital CMV is the most common infectious cause of deafness in the United States.
B. FALSE: Women infected at least 6 months prior to pregnancy are far less likely to deliver babies who suffer from
congenital CMV infection.
C. FALSE: Congenital CMV is the most common congenital infection.
D. FALSE: Congenital CMV is the most common infectious cause of intellectual disability in the United States.
61. A 54-year-old female presents with the oral findings below. Additional physical exam findings included flat-
topped pink papules on the wrist and complaints of mild difficulty swallowing. Which additional workup item
should be considered for this patient?
A. Pap smear
B. Upper endoscopy
C. Colonoscopy
D. Abdominal ultrasound
E. Patch testing
ANSWER: B.
EXPLANATIONS:
B. TRUE: The esophagus is the most frequently involved gastrointestinal site of lichen planus (LP). Although the
presentation is rare, LP can be an important potential cause of morbidity. The presence of oral LP along with
reported symptoms such as dysphagia require further workup to rule out esophageal strictures. If present, the
strictures typically involve the mid or upper esophagus. The presence of esophageal LP increases the risk of
developing squamous cell carcinoma, and its presence is not always symptomatic, making surveillance for
esophageal involvement important to consider.
A. FALSE: Pap-smears are not indicated in oral LP. For variants of LP where the vaginal introitus is involved, there is
an increased risk of developing squamous cell carcinoma. This is a significant concern in the ulcerative or erosive
variants of vulvovaginal LP. When necessary, biopsies of the involved areas are necessary in order to rule out
squamous cell carcinoma.
C, D, and E. FALSE: These options are not necessary for the management of this patient.
28
62. A 45-year-old male presents to the ED with a persistent rash on his palms and soles.The lesions are tan/pink
colored macules. He also has a small erythematous and white plaque on his right lateral tongue. A diagnosis of
syphilis is suspected.Which of the following forms of syphilis is incorrectly matched with its time course after
primary infection?
A. Primary syphilis – 10 to 90 days
B. Secondary syphilis – 5 to 23 weeks
C. Tertiary syphilis – 18 weeks to 15 years
D. Early latent syphilis – <1 year
E. Late latent syphilis – >1 year
63. You are asked to examine the skin of an infant born to a mother who experienced an infection with herpes
simplex virus for the first time during her second trimester. The birth was vaginal, and a close examination
revealed no mucosal lesions in the birth canal, and no prodromic symptoms were suggestive of HSV infection
prior to and during delivery. Concern is raised about an intrauterine HSV infection. Which of the following
findings is unlikely to be present in this newborn during the neonatal period?
A. Microcephaly
B. Cutaneous lesions
C. Chorioretinitis
D. Seizures
E. Sepsis
ANSWER: E.
EXPLANATIONS:
E. TRUE: Infants who acquire infection with the herpes simplex virus (HSV) in utero, also known as an intrauterine
HSV infection, do not demonstrate clinically the signs and symptoms of sepsis or systemic toxicity. This is in
contrast to neonates who acquired HSV infection during the perinatal or postnatal period from mothers who
experienced primary genital HSV infection.
A, B, C, and D. FALSE: Cutaneous and central nervous system involvement can be present in the neonatal period,
present as late sequelae, or frequently appear at the time of birth. Chorioretinitis is present in 60% of infants born
with intrauterine HSV infection, and microcephaly is present in 50% of patients who acquired an HSV infection in
utero.
29
64. A 6-year-old male presents to the emergency room with pruritic papules a few hours following a visit to the
beach. There is no evidence of other mucocutaneous lesions. Which of the following is the most likely cause of
this condition?
ANSWER: C.
EXPLANATIONS:
C. TRUE: This small jellyfish, with a size around 1.5 cm in diameter, is common in the waters of the Caribbean
(Belize, Cuba, Mexico), Gulf of Mexico, and tropical North Atlantic West (Florida, Cuba and Bahamas). When the
bather leaves the water, the suit acts as a filter, draining the water and imprisoning the larvae against the skin
while activating the defense mechanism of these jellyfish and causing toxin release.
A. FALSE: Herpes simplex virus is not a cause of pruritic papules in this distribution.
B. FALSE: The distribution of this rash is atypical for dermatitis herpatiformis.
D. FALSE: Cercarial dermatitis, also known as swimmer's itch, is caused by a cutaneous infestation by the larval
stage of trematodes of the genera Schistosoma and Trichobilharzia. This eruption typically occurs from freshwater
exposure to uncovered areas on the body.
65. In a 20-weeks pregnant woman with known exposure to parvovirus B19, which of the following is the next step
in management?
A. Serial fetal ultrasonography
B. Amniocentesis
C. Chorionic villus sampling
D. Serologic testing for IgM and IgG antibodies
E. Maternal and fetal serum PCR studies
ANSWER: D.
EXPLANATIONS:
D. TRUE: In a pregnant woman with exposure to parvovirus B19, the next step in management is serologic testing
for IgM and IgG.
A. FALSE: If testing is positive, serial fetal ultrasonography is indicated.
B. FALSE: While some studies have shown amniotic fluid sampling beneficial in the detection of parvovirus B19, this
method of viral detection would not be indicated unless other studies were insufficient.
C. FALSE: Chorionic villus sampling is not part of the management in suspected parvovirus B infection of pregnant
women.
E. FALSE: Occasionally, yield from serologic testing for IgG and IgM is inconclusive, and maternal and fetal serum
PCR studies may be beneficial to confirm the diagnosis. However, these studies are not first-line
30
66. A 6-year-old child had a recent history of low-grade fevers and cold-like symptoms. A few days later, the patient
developed an erythematous rash on the cheeks and an erythematous lacy rash on the torso and proximal
extremities. Of note, a peripheral blood smear from several years prior showed unusually small erythrocytes
lacking normal central pallor. What condition must the practitioner be aware of as a risk in this patient?
A. Acute leukemia
B. Acute splenomegaly
C. Acute kidney failure
D. Aplastic crisis
E. Purpuric vasculitis
ANSWER: D.
EXPLANATIONS:
D. TRUE: This is a child with parvovirus B19 infection (fifth disease). The disease is generally not harmful long-term
in otherwise healthy patients. However, patients with chronic anemic states such as sickle cell anemia or
hereditary spherocytosis (as in this case) are at risk for aplastic crisis.
67. A 73-year-old man presents to the clinic with recent-onset tinnitus. He is also having slight difficulty moving the
right side of his face. He states that about 3 weeks ago, he developed a painful rash around his right ear. He
noted pain in the area followed by the development of "red bumps" that eventually turned into "watery
bumps" and crusted over. Which part of this patient’s nervous system is likely affected?
A. Otic ganglion
B. Trigeminal ganglion
C. Geniculate ganglion
D. Scarpa’s ganglion
E. Pterygopalatine ganglion
ANSWER: C.
EXPLANATIONS:
C. TRUE: There are 3 types of Ramsay Hunt syndrome. The type caused by reactivation of varicella is Ramsay Hunt
type II. In this syndrome, reactivation occurs and affects the geniculate ganglion. This can result in ipsilateral facial
nerve palsy/paralysis, ear pain, vertigo, changes in gustatory sensation, and alterations in hearing such as hearing
loss and tinnitus. Some of the otic alterations are because the geniculate ganglion lies in such close proximity to
the vestibulocochlear nerve.
68. A 14-year-old girl is evaluated at the pediatric clinic due to light and dark patches of the skin that have been
present for 2 months and are not pruritic. She is taking no medications and had no unusual skin exposures.
Examination reveals a young girl of a medium complexion with 12 slightly scaly patches on the chest, back, and
upper abdomen. Most are hypopigmented, but a few show an increased reddish-brown hue. Woods' light
examination demonstrates an orange fluorescence. To confirm your clinical suspicion, what is the next step?
A. Perform a skin biopsy
B. Obtain a lupus (LE) prep
C. Perform a KOH prep
D. Order VDRL testing
E. Obtain ANA titers
31
Correct Answer: C. Perform a KOH prep
EXPLANATIONS:
Hypopigmented and hyperpigmented non-pruritic macules and/or patches that fluoresce under the Woods' lamp
strongly suggest tinea versicolor. This condition is caused by a superficial (stratum corneum) skin infection by fungi
of the Malassezia genus, most commonly of the globosa and furfur species, which are normal skin inhabitants.
With the conversion from spores to hyphae (mycelia stage), the clinical picture of tinea versicolor occurs. The
condition is most common from the teenage years to young adulthood. A KOH preparation reveals the so-called
"spaghetti and meatballs" (hyphae and spore) pattern. The pigmentary changes are not permanent. A skin biopsy
is unnecessary. Unlike vitiligo, which produces permanent hypopigmentation (without hyperpigmentation), tinea
versicolor does not have an autoimmune basis. The lesions are not characteristic of secondary syphilis.
69. A 34-year-old female was seen in the hospital by the dermatology service for an evaluation of erythematous
targetoid lesions, necrotic nodules, and abscesses. A biopsy showed 45-degree branching hyphae. A fungal
culture grew Fusarium. Which of the following is incorrect concerning Fusarium?
A. Fusarium is seen more often in samples of air than Aspergillus.
B. Fusarium is more likely to occur in patients with hematologic malignancies.
C. Fusarium is the most common fungal infection in burn patients.
D. The treatment of choice is amphotericin B and flucytosine.
E. Fatality reaches approximately 50% in patients with prolonged severe neutropenia.
ANSWER: E.
EXPLANATIONS:
E. This is the "incorrect" statement. Fatality of patients with severe prolonged neutropenia reaches approximately
100%. The other options all represent correct statements.
A. Fusarium is found more often in air samples than Aspergillus.
B. Patients with heme malignancies are predisposed to Fusariosis. Other groups include those with neutropenia,
trauma, or burns.
C. This is the most common fungal infection in burn patients. Aspergillosis is the second most common.
D. Treatment for Fusariosis includes amphotericin B and flucytosine.
70. A 7-year-old patient is seen in the clinic on Monday morning after being sent home from school for the rash
depicted below. The child had a flu-like illness the previous week, characterized by a fever, a headache,
myalgia, and malaise. His mother reports that his face appeared flushed over the weekend, with bright red
cheeks, but this has resolved. Which of the following statements describes this scenario?
32
ANSWER: A.
EXPLANATIONS:
A. TRUE: The image illustrates the lacy reticular pattern that follows the “slapped cheek” appearance of erythema
infectiosum. This presentation is a common clinical presentation following infection of parvovirus B-19. Following the
initial infection, a mild prodrome consisting of a low-grade fever, headaches, and myalgias occurs 7-10 days before
the classic erythema of the cheeks. The appearance of the rash marks the antibody isotype switching of IgM to IgG
and also signifies a conclusion of the infectious state. Following initial inoculation and during the prodrome, while
viremia takes place, the predominant antibody is IgM (a marker of the infectious state).
B. FALSE: The infection is transmitted via respiratory droplets and not through bloodborne products.
C and D. FALSE: The onset of cutaneous findings and arthralgias is an indicator that this period of infectivity is no
longer present. Parvovirus B19 has a tropism for erythroid progenitor cells. While in normal individuals it has a limited
clinical severity, transmission to individuals with hemolytic abnormalities as well as women during pregnancy may
lead to significant complications such as aplastic crisis, pancytopenia, and more significantly hydrops fetalis.
71. A 42-year-old man who recently immigrated from Africa presents to the clinic with a 4 cm ulcer on his right
forearm. The erythematous/yellow thin ulcer has an erythematous raised border. He remembers being bitten
by "some type of fly" about 2 months ago. A biopsy is performed, and a rod-shaped kinetoplast is observed
within the amastigote. Which of the following is incorrect concerning this condition?
A. The vectors are Phlebotomus (old world) and Lutzomyia (new world).
B. The causative organism is cultured on Novy-MacNeal-Nicolle medium
C. Treatment is with Pentavalent antimony.
D. New-world causative organisms include L. tropica, L. major, and L. infantum.
E. L. donovani is an old-world organism that causes the visceral form of the disease.
ANSWER: (D) New-world causative organisms include L. tropica, L. major, and L. infantum.
Explanation:
L. tropica, L. major, and L. infantum (along with L. aethiopica and L. donovani) are all old-world Leishmania.
Specimens used in diagnosis should be collected from lesions that are active and ulcerative that have no evidence of
secondary infection. The Leishmania amastigote is an oval to round organism that is 1.5 to 4.0 microns in diameter
with a cell membrane, cytoplasm, internal nucleus, and rod-shaped kinetoplast containing mitochondrial DNA. On
Giemsa staining, the kinetoplast appears red or violet.
Incorrect answers:
(A) The sandfly species that serve as vectors for Leishmaniasis are Phlebotomus (old world) and Lutzomyia (new
world).
(B) Novy-MacNeal-Nicolle medium is used to culture Leishmania.
(C) Pentavalent antimony is the treatment of choice for Leishmaniasis.
(E) L. donovani causes visceral disease in India and Kenya.
72. Which of the following scenarios warrants eradication of Staphylococcus carriage through twice daily
application of mupirocin to the nares for 5-10 days?
Non-bullous impetigo
Bullous impetigo
Recurrent folliculitis
Gram-negative folliculitis
Hot-tub folliculitis
33
ANSWER: C.
EXPLANATIONS:
C. TRUE: Recurrent folliculitis due to Staphylococcus aureus is a sign of possible chronic carriage. Eradicating nasal
carriage through the application of twice daily mupirocin for 5-10 days for the patient and close contacts may lead
to resolution of folliculitis with lowered rates of recurrence.
A and B. FALSE: Both bullous and non-bullous impetigo are localized, toxin-mediated skin infections, and their
presence is not a sign of chronic staph carriage. However, in patients who have recurrent impetigo despite
adequate and appropriate treatment, bacterial cultures can be taken from the nares in order to rule in or out
chronic carriage. The patient is then treated as described in the vignette.
D. FALSE: Gram-negative folliculitis is a result of chronic and long-term antibiotic use. It is often seen in patients
with moderate-to-severe acne who have been on long courses of oral tetracyclines.
E. FALSE: Hot-tub folliculitis results from the improper chlorination and decontamination of hot-
tubs/jacuzzis/whirlpool baths and is due to Pseudomonas infection.
73. A 28-year-old male graduate student reports with severe malaise and a cough. He is experiencing night sweats
and has had minor weight loss. He recently visited caves in Mexico to collect specimens for his lab work. His
illness started about 3 days after his return. A diagnosis of histoplasmosis is suspected. Of the following, which
fungus is incorrectly matched with its histological description?
A. Histoplasma capsulatum - intracellular 2-4 micrometer budding or singular cells with a
capsule
B. Blastomyces dermatitidis - granuloma reaction with 8-15 micrometer broad-based budding
cells
C. Coccidioides immitis - 20-80 micrometer spherules
D. Paracoccidioides brasiliensis - 5-50 micrometer round cells with multiple buds attached to a
main cell through a thin connection
E. Aspergillosis sp. - hyphae branching at 45-degree angles with small round conidia
ANSWER: A.
EXPLANATIONS:
A. TRUE: This is incorrectly matched. H. capsulatum is a 2-4 micrometer budding or singular cell, but it does not
have a capsule. It has a surrounding halo that is merely a histological artifact.
B. FALSE: Blastomyces dermatitidis does demonstrate a granuloma reaction with 8-15 micrometer broad-based
budding cells.
C. FALSE: Coccidioides immitis consists of 20-80 micrometer spherules.
D. FALSE: Paracoccidioides brasiliensis consists of 5-50 micrometer round cells with multiple buds attached to the
main cell through a thin connection (Mariner's wheel).
E. FALSE: Aspergillosis sp. demonstrates hyphae branching at 45-degree angles with small round conidia.
74. Which of the following neonatal herpes simplex virus (HSV) infections warrants further investigative studies to
rule out central nervous system disease?
A. Infection localized to the skin, eyes, or mouth
B. Disseminated infection
C. Encephalitis without skin, eye, or mouth involvement
D. Encephalitis with skin, eye, or mouth involvement
E. All of the above
34
ANSWER: E.
EXPLANATIONS:
E. TRUE: Any newborn with skin or mucosal lesions of HSV, regardless of presenting signs or symptoms, should
undergo the necessary testing to rule out underlying disseminated or central nervous system disease. HSV infection
of the skin, eyes, and mouth (aptly named SEM disease) is how approximately 40% of cases of neonatal HSV
infection present. Cutaneous lesions appear vesicular and herpetiform in nature and typically are present at the
sites of prolonged contact with the cervix. The site of the scalp electrode is a common location for cutaneous
lesions of HSV infection. Scars on the skin and oral mucosa may be present at birth, representing residua of pre-
existing HSV vesicles. The skin/mucosal involvement in SEM disease alone is not enough to preclude additional
testing to rule out disseminated or CNS disease. Both disseminated neonatal HSV infection and CNS disease due to
HSV infection commonly present with cutaneous findings in 77% and 60% of patients, respectively.
Disseminated disease due to HSV infection has a mortality rate approaching 75% of patients without specific
antiviral therapy. The multisystem involvement in disseminated disease is analogous to the presenting signs and
symptoms of overt bacterial sepsis with shock, disseminated intravascular coagulation, and organ system failure
as common findings. Even with specific antiviral therapy, mortality remains high. In survivors, delayed neurological
sequela is a common complication. As in the case of SEM disease, cutaneous findings are often the first sign of
neonatal infection. In contrast to other forms of neonatal HSV infection, HSV encephalitis may take 2-3 weeks to
manifest. Symptoms present subtly with increased irritation and agitation before they slowly progress to seizures.
Cutaneous lesions may be initially present in approximately 50-60% of patients with HSV encephalitis, making this
form of neonatal HSV difficult to distinguish initially. Therefore, any presentation of neonatal HSV infection
requires additional workup, examination, and close follow-up to rule out systemic infection.
75. All of the following are complications of ulcerative vulvovaginal lichen planus EXCEPT?
A. Dyspareunia
B. Scarring
C. Squamous Cell Carcinoma
D. Post-coital bleeding
E. Obliteration of the clitoral hood
35
76. A 36-year-old female presents with a new generalized morbilliform rash. Two weeks prior, she experienced a
high fever, malaise, myalgias, and a sore throat and was given antibiotics by her primary care physician. Later,
she developed the current morbilliform rash. What infection did she most likely suffer from, and what
antibiotic did she receive?
EBV infection - amoxicillin
CMV infection - clindamycin
Streptococcal infection - amoxicillin
EBV infection - clindamycin
CMV infection – amoxicillin
ANSWER: A.
EXPLANATIONS
A. TRUE: Infectious mononucleosis can often mimic symptoms of bacterial pharyngitis (e.g. fever, sore throat, and
malaise). While a minority of patients may present with a rash during EBV infection, almost all patients with active
EBV infection given amoxicillin or ampicillin will develop a generalized pruritic morbilliform eruption.
77. A Colombian female presented to the emergency room with a recent onset of myalgias and abdominal pain.
What is the most likely mode of transmission?
ANSWER: A.
EXPLANATIONS:
A. TRUE: The image displays the clinical appearance of Romaña’s sign, a marker of acute Chagas disease. This is
caused by protozoal infection with Trypanosoma cruzi. These organisms typically infest the GI tract of triatomine
vectors, which subsequently take blood meals from humans and transmit the organism.
B. FALSE: A. aegypti is the vector for dengue fever, not Chagas disease.
C. FALSE: Although ingestion of large amounts of T. cruzi may cause possible infection, this is not the most common
cause.
D. FALSE: This is not a sexually transmitted disease.
E. FALSE: Blood transfusions may transmit Chagas disease; however, the most common mode of transmission is via
triatomine vectors.
78. A 6-year-old child who recently immigrated from Eastern Europe presents with a recent mild flu-like illness
and a rash consisting of pink petechiae and macules. Small erythematous petechiae are noted on the soft palate.
Per the child’s mother, the rash first appeared on the face and spread. As it spread to the body, the rash on the
face resolved. What type of virus is the causative agent of this condition?
A. Poxvirus
B. Parvovirus
C. Togavirus
D. Orthomyxovirus
E. Picornavirus
36
ANSWER: C.
EXPLANATIONS:
C. TRUE: The clinical vignette describes a child with rubella, which typically begins with mild flu-like symptoms and
a pink to an erythematous petechial/macular rash that begins on the face, clearing as it spreads to other parts of
the body. The rash resolves after a few days. The illness is caused by the rubella virus, which belongs to the
Togavirus family. The virus is spread by the respiratory route. After an incubation period of 1 week, the flu-like
symptoms and rash begin. Forchheimer’s sign consists of small red macules and petechiae that appear on the soft
palate. These do not occur in all cases of rubella.
ANSWER: A.
EXPLANATIONS:
A. TRUE. Impetigo is a highly contagious infectious bacterial disease and is transferred either person-to-person or
through contact with fomites.
B. FALSE. The highest incidence of impetigo is in children under 6 years of age.
C. FALSE. Peak incidence for impetigo is in the summer months with warm ambient temperature and high humidity
being predisposing factors.
D. FALSE. Contact sports such as wrestling, football, and boxing increase the risk of acquiring and transmitting
Staphylococcus aureus and therefore increase the risk of acquiring impetigo.
E. FALSE. Staphylococcus aureus is the leading cause of both bullous and non-bullous impetigo. Streptococcus
pyogenes may be a cause in more temperate climates, but overall S. aureus is more commonly isolated.
ANSWER: C.
EXPLANATIONS:
C. TRUE: Chikungunya fever is caused by the chikungunya virus, which is a togavirus.
A. FALSE: Variola virus is an orthopoxvirus that causes smallpox.
B. FALSE: Ecthyma contagiosum is caused by the Orf virus, a parapoxvirus.
D. FALSE: Vaccinia is caused by the vaccinia virus, an orthopoxvirus.
E. FALSE: Milker’s nodules are caused by Paravaccinia/Parapox infection.
81. Which of the following diseases has been associated with polyomavirus?
A. Trichodysplasia spinulosa
B. Butcher’s wart
C. Dengue fever
D. Gianotti-Crosti syndrome
E. Ecthyma gangrenosum
37
ANSWER: A.
EXPLANATIONS:
A. TRUE: Viral-associated folliculodystrophy has been associated with polyomavirus infection in immunosuppressed
patients, including leukemia/lymphoma patients receiving chemotherapy and solid-organ transplant patients.
B. FALSE: Butcher’s nodules are caused by HPV-7 infection.
C. FALSE: Dengue fever is caused by 1 of 4 subtypes of dengue virus, all Flaviviridae.
D. FALSE: Gianotti-Crosti is associated with multiple viral infections, but polyomavirus has not been reported in
association with Gianotti-Crosti syndrome.
E. FALSE: Ecthyma gangrenosum is caused by Pseudomonas aeruginosa bacteremia.
82. A 26-year-old male recently returned from traveling to Brazil. Once home, the patient developed skin lesions
and diarrhea. Which of the following organisms likely causes diarrhea in association with the cutaneous
findings pictured below?
A. Streptococcus pyogenes
B. Staphylococcus epidermidis
C. Echinococcus granulosus
D. Onchocerca volvulus
E. Strongyloides stercoralis
ANSWER: E.
EXPLANATIONS:
E. TRUE: Strongyloides stercoralis causes a distinctive form of cutaneous larva migrans that is recognized as larva
currens. Larva currens presents with serpiginous erythematous papules and subsequent hypopigmentation on the
buttocks, upper thighs, and lower abdomen. Perianal involvement or hematogenous spread in an immunosuppressed
patient can lead to intestinal infestation resulting in diarrhea.
A. FALSE: S. pyogenes infection can cause skin manifestations such as impetigo and ecthyma, but it is not a cause of
this distinctive serpiginous pattern.
B. FALSE: E. epidermidis is normal skin flora that is unlikely to cause a skin infection in an immunocompetent patient.
C. FALSE: In echinococcosis, the liver and lungs are the major sites of involvement, with the development of 1 or more
hydatid cysts. Under 2% of patients develop soft tissue cysts, presenting as firm subcutaneous nodules or as masses
that are fluctuant and non-tender.
D. FALSE: Skin manifestations of onchocerciasis include subcutaneous nodules containing adult worms, pruritic
papular dermatitis, lichenification, and leukoderma.
83. Which Bartonella species is incorrectly matched with the disease it causes?
A. B. henselae – cat-scratch disease
B. quintana – bacillary angiomatosis
C. quintana – trench fever
D. B. henselae – verruga peruana
E. B. bacilliformis – carrion’s disease
38
ANSWER: D.
EXPLANATIONS:
D. This answer is correct. Verruga peruana is the chronic form of Oroya fever, caused by B. bacilliformis.
A. Cat-scratch disease is caused by B. henselae.
B. Bacillary angiomatosis is caused by B. quintana.
C. Trench fever is caused by B. quintana.
E. Carrion’s disease is the acute form of Oroya fever, caused by B. bacilliformis.
84. A 9-month-old male is hospitalized for exquisitely tender ulcers on the extremities with associated fevers, chills,
and poor appetite. Which of the following statements is incorrect regarding the ulcerative lesions shown?
ANSWER: A.
EXPLANATIONS:
A. TRUE: The figure displays the typical appearance of ecthyma gangrenosum. Primarily affecting immunocompromised
patients, severe neutropenia (not neutrophilia) represents a major predisposing factor.
B and C. FALSE: Associated with P. aeruginosa septicemia, a biopsy shows a necrotizing hemorrhagic vasculitis of deep
vessels, with gram-negative rods occasionally seen in the medial and adventitial wall. The intima is usually spared.
D. FALSE: Lesions are most commonly located in the anogenital area or on the extremities.
E. FALSE: IV antipseudomonal antibiotics are the mainstay of therapy in ecthyma gangrenosum.
85. A 27-year-old HIV-positive man presents with a 3 cm erythematous lesion with a central necrotic ulcer on his
right dorsal forearm. A diagnosis of ecthyma gangrenosum is suspected. What is the pathophysiology of the
formation of this patient’s lesion?
A. Infection of the epidermis and superficial dermis
B. Embolic event
C. Perivascular invasion by an organism with the release of exotoxin
D. Invasion of bacteria to the vessel lumen
E. Destruction of the dermis and subcutaneous tissue by a bacterial endotoxin
ANSWER: C.
EXPLANATIONS:
C. TRUE: The causative organism of ecthyma gangrenosum is Pseudomonas aeruginosa. The bacteria invade the
perivascular space and secrete an exotoxin, which causes local tissue destruction.
39
86. A 61-year-old man from North Carolina presents to the emergency room with a 5-6 day history of persistent
fevers, headaches, severe pain, and cytopenias following a tick bite. The patient denies recent travel. On
physical examination, a petechial rash involving the palms and soles is noted. Labs are drawn and pending.
What is the most likely species of tick associated with this clinical scenario?
A. Amblyomma cajennense
B. Ixodes scapularis
C. Coxiella burnetii
D. Dermacentor variabilis
E. Leptotrombidium delicense
ANSWER: D.
EXPLANATIONS:
D. TRUE: This clinical scenario exhibits a typical clinical scenario for RMSF. Location typically determines the
responsible vector. In the United States, Rickettsia rickettsii, the causative organism of RMSF, is transmitted to
humans by several tick species. In the Eastern, Central, and Pacific Coastal United States, the species that most
frequently transmits R. rickettsii is the American dog tick (Dermacentor variabilis). Dermacentor andersoni (the
Rocky Mountain wood tick) is a frequent cause in the western United States.
A. FALSE: The cayenne tick (Amblyomma cajennense) is a common vector for Rocky Mountain spotted fever (RMSF)
in Central and South America. It has been rarely reported in the United States.
B. FALSE: Ixodes scapularis is the vector responsible for a number of diseases, including Lyme disease and
anaplasmosis. It is, however, not a common vector for RMSF.
C. FALSE: Coxiella burnetii is the causative organism of Q fever.
E. FALSE: L. deliense is a vector and reservoir for scrub typhus endemic to Asia. This is also not a vector for RMSF.
87. An intubated patient in the intensive care unit has a suspected hypersensitivity reaction. The patient’s family
reveals that the patient has a known penicillin allergy. While reviewing the list of the patient’s current
medications, the clinician notes that no penicillins are listed, but the patient is receiving a cephalosporin. Which
of the following is a fourth-generation cephalosporin?
A. Cefepime
B. Cephalexin
C. Cefaclor
D. Cefotaxime
E. Ceftriaxone
ANSWER: A.
EXPLANATIONS:
A. TRUE: Cefepime is a fourth-generation cephalosporin. Fifteen percent of people with penicillin allergy have
cross-reactivity to cephalosporins. Also, 2% of people with a cephalosporin allergy have cross-reactivity to
penicillins. Cephalosporins and penicillins may cause similar cutaneous findings, including morbilliform eruption,
urticaria, drug fever, eosinophilia, and acute generalized exanthematous pustulosis.
B. FALSE: Cephalexin is a first-generation cephalosporin, along with cefadroxil and cefazolin.
C. FALSE: Cefaclor is a second-generation cephalosporin along with cefprozil, cefuroxime axetil, cefuroxime,
cefotetan, and cefoxitin.
D. FALSE: Cefotaxime is a third-generation cephalosporin along with cefixime, cefdinir, cefpodoxime, ceftazidime,
ceftibuten, cefditoren, and ceftriaxone.
E. FALSE: Ceftriaxone is a third-generation cephalosporin.
40
88. Which of the following pairs of human papillomavirus (HPV) type and related condition is incorrect?
A. HPV 32 – Epidermodysplasia verruciformis
B. HPV 11 – Bushke-Lowenstein tumor
C. HPV 16 – High-risk anogenital/cervix lesions
D. HPV 1 – Palmoplantar wart
E. HPV 7 – Butcher’s wart
ANSWER: A.
EXPLANATIONS:
A. TRUE: HPV 32 (and 13) causes oral focal epithelial hyperplasia. The following types cause Epidermodysplasia
verruciformis: 5, 8, 9, 12, 14, 15, 17, and 19-26.
B. FALSE: HPV 11 causes Bushke-Lowenstein tumor and anogenital lesions.
C. FALSE: HPV 16, 18, 31, 33-35 cause high-risk anogenital lesions.
D. FALSE: HPV 1 causes palmoplantar warts.
E. FALSE: HPV 7 causes butcher’s warts.
89. IV acyclovir is the first-line therapy for which of the following presentations of neonatal herpes?
A. Disseminated herpes simplex virus (HSV) infection
B. Skin, eyes, and mouth HSV infection
C. Herpes encephalitis
D. All of the above
E. A and C only
ANSWER: D.
EXPLANATIONS:
D. TRUE: Because both the disseminated variant and the CNS disease variant of neonatal herpes virus infection can
initially present with only cutaneous or mucosal skin lesions, intravenous acyclovir is indicated as the first-line
therapy in all of the listed presentations of neonatal herpes. Up to 60% of patients who will eventually develop
HSV encephalitis (a characteristic of CNS disease) will present with similar cutaneous findings to patients with
neonatal HSV infection confined to the skin and mucous membranes. For patients with disseminated disease,
however, systemic signs of sepsis, shock, and end-organ damage would require a more aggressive approach,
including the initiation of IV acyclovir.
90. A 12-year-old girl presents to the pediatric clinic with a recent onset of illness. Her symptoms include a sore
throat and a fever. She also has an erythematous fine rough rash, which has worsened and spread over several
days. Which of the following is true regarding this condition?
A. The incubation period is usually short, lasting only a few days.
B. Forchheimer spots, seen on the tongue and gums, are signs of this disease.
C. The typical rash spares folds of skin, creating Pastia lines.
D. This illness is mediated by pyogenic exotoxin E.
E. The rash typically appears on the dorsal hands and spreads to other parts of the body.
ANSWER: A.
EXPLANATIONS:
A. TRUE: The clinical vignette is describing scarlet fever, typified by a fine rough sandpaper-like rash. The incubation
period is typically 1 to 4 days.
B. FALSE: Forchheimer spots are associated with rubella but are also seen in measles and scarlet fever. They appear
on the soft palate, not the tongue or gums. This patient may present with a "strawberry" tongue.
C. FALSE: Pastia lines are folds of skin where the typical rash is accentuated (not spared).
41
D. FALSE: The mediators are pyogenic exotoxins A, B, and C.
E. FALSE: The typical rash starts around the head and neck and spreads to other parts of the body.
91. A 54-year-old sheep farmer presents with multiple firm red papules on the bilateral phalanges that gradually
enlarged over the past week to become pustular. What is the best treatment for this patient?
A. Acyclovir
B. Ciprofloxacin
C. Doxycycline
D. Watch and wait
E. Cimetidine
ANSWER: D.
EXPLANATIONS:
D. TRUE: The presence of pustules on an upper extremity in a sheep farmer is suggestive of orf. Orf is caused by a
parapoxvirus endemic in sheep and goats. It is transmitted to humans through direct contact. Supportive care is
the most appropriate intervention since orf typically self-resolves within 4 weeks. Systemic findings such as
lymphangitis and fever are rare and typically mild.
A. FALSE: Acyclovir is used to treat herpes virus infection. Lesions of herpesvirus typically have the appearance of
grouped vesicles, the vesicular stage of orf can mimic herpetic whitlow.
B and C. FALSE: Anthrax can also infect those in contact with sheep and is on the differential diagnosis for orf. The
lesion of cutaneous anthrax is more hemorrhagic with rapid progression to a necrotic eschar. The treatment for
cutaneous anthrax is antibiotics such as doxycycline or ciprofloxacin.
E. FALSE: Cimetidine is an H2 antagonist that can be used to treat molluscum contagiosum, a pox virus, with mixed
results. Molluscum presents as clusters of small papules with central umbilication.
92. A 54-year-old female presents to your clinic with a complaint of pain and itchiness of her scalp. She is mostly
concerned with the localized hair loss illustrated below. Which of the following is the most likely diagnosis?
A. Androgenic alopecia
B. Lichen planopilaris
C. Syphilitic alopecia
D. Dissecting cellulitis of the scalp
E. Alopecia areata
42
ANSWER: B.
EXPLANATIONS:
B. TRUE: The areas of the scalp seen in the photo that are devoid of any remnant hair follicles categorize this
condition as a scarring alopecia. A telling feature that will aid the diagnosis is the perifollicular erythema
associated with the scarring alopecia. This clinical finding is consistent with the diagnosis of lichen planopilaris.
A and E. FALSE: Androgenic alopecia and alopecia areata may present with patchy hair loss and do not have follicle
unit loss and scarring as a feature.
C. FALSE: Syphilitic alopecia, which presents with a more diffuse and patchy presentation, could be considered but
rarely leads to the scarring seen in the photograph.
D. FALSE: While dissecting cellulitis of the scalp is a scarring alopecia, it is characterized by boggy and indurated
plaques of alopecia scattered over the scalp. They may present with some mild discomfort, but this is often
overshadowed by the foul-smelling discharge that can often accompany this condition.
93. The presence of the lesions illustrated below warrants which of the following laboratory tests?
A. Hepatitis panel
B. Anti-nuclear antibody panel
C. Complete blood count
D. Comprehensive metabolic panel
E. Liver function tests
ANSWER: A.
EXPLANATIONS:
A. TRUE: Of the many types of lichen planus (LP), the oral variant is most frequently viewed as a manifestation of
hepatitis C virus infection. In one study, PCR techniques found HCV RNA in 93% of oral lichen planus lesions.
However, it is unclear if there is a genetic predisposition to oral LP and HCV has a role as a trigger in this
population or if HCV is a causal factor initiating the onset of LP.
B. FALSE: While lichen planus/lupus erythematosus overlap syndrome exists, lesions associated with it
predominantly involve acral sites and spare mucous membranes.
C, D, and E. FALSE: These measures are not necessary for the workup or management of oral lichen planus.
94. A 9-year-old female is admitted for recurrent staphylococcal cellulitis. On exam the patient is noted to have
numerous verrucous papules and plaques, including perianally. Lab workup reveals low levels of IgG. Which of
these is likely to be true for this patient?
A. CBC would show neutrophilia
B. Mature neutrophils in the bone marrow
C. The patient is likely to have a history of sexual abuse
D. A mutation in the CCR5 gene
E. The patient has primary lymphedema
43
Correct answer: (B) Mature neutrophils in the bone marrow.
Explanation: This is an example of WHIM syndrome. This is a rare autosomal dominant immunodeficiency
characterized by Warts, Hypogammaglbulinemia, Infections (recurrent bacterial), and myelokathexis (neutropenia
due to retention of neutrophils in the bone marrow).
(A) The CBC would show neutropenia due to myelokathexis.
(C) Patient are susceptible to HPV and the condylomata are typically not associated with sexual abuse.
(D) WHIM syndrome is caused by a defect in the CXCR4 gene.
(E) Primary lymphedema is found in WILD syndrome: Warts, Immunodeficiency, Lymphedema (primary) and
multifocal anogenital Dysplasia.
95. You are asked to see a 2-day-old neonate for a rash consisting of grouped vesicles on the scalp, face, and lateral
neck. You are unable to retrieve an accurate medical history from the mother regarding previous infections
and are concerned about neonatal herpes virus infection. You enter orders for intravenous acyclovir for the
patient. Which of the following would not be prevented by this therapeutic modality?
A. Dissemination of cutaneous lesions
B. Mucosal ulcerations
C. Central nervous system involvement
D. Chorioretinitis
E. Acute liver failure
ANSWER: C.
EXPLANATIONS:
C. TRUE: Central nervous system disease in neonatal HSV infection presents with encephalitis. Unlike the disseminated
or skin, eyes, and mouth (SEM) disease variants of neonatal HSV infection, CNS disease may take several weeks to
present (mean age of 17 days). To complicate diagnosis and management further, 40% of patients who have a
diagnosis of HSV encephalitis present with cutaneous findings similar to that of the SEM and disseminated variant.
Neurologic sequelae are typically not impeded by the initiation of appropriate antiviral therapies such as IV acyclovir.
Despite early therapeutic intervention, neurologic and developmental outcomes remain abnormal.
A, B, D, and E. FALSE: These are all systemic and cutaneous complications of neonatal HSV infection that can be
treated and prevented with early initiation of an appropriate antiviral regimen.
96. A 38-year-old woman who recently immigrated from Mexico presents with her 16-year-old daughter acting as
translator. The patient’s daughter was worried about several diffuse patches of hypopigmented skin around her
mother’s face, arms, and legs. The patient notes more patches on her back and buttock areas. In the areas of
hypopigmentation, her hair had “fallen out and stopped growing.” On physical exam, the patches are clearly
demarcated with a raised erythematous border and scaly-looking centers. When asked when her rash
appeared, the patient states that approximately 4 years ago, she began feeling painful tingling in her arms and
that the rash appeared in those areas months later. A biopsy of a lesion on her arm reveals acid-fast bacilli and
granuloma formation within dermal nerves. What is the most likely causative agent?
A. Treponema pallidum pallidum
B. Mycobacterium leprae
C. Treponema pallidum carateum
D. Treponema pallidum pertenue
44
ANSWER: B.
EXPLANATIONS:
B. TRUE: Mycobacterium species of bacteria are acid-fast, and this clinical presentation describes leprosy. The
patchy rash in leprosy is described as “white spots” or areas of hypopigmentation that lack hair follicles or sweat
glands. Granulomas within dermal nerves on a lesion biopsy are diagnostic for Mycobacterium leprae.
A. FALSE: Treponema pallidum subspecies pallidum is a spirochete bacteria and the causative agent of syphilis.
Because these bacteria are so small, they do not stain acid-fast and can only be visualized using light microscopy
and dark-field illumination.
C and D. FALSE: T. pallidum subspecies pertenue causes yaws, while carateum causes pinta.
ANSWER: B.
EXPLANATIONS:
B. TRUE: The Epstein-Barr virus is a DNA virus. The Togavirus group, of which Rubella virus is a member, includes
RNA viruses. The human papillomavirus is a DNA virus and causes viral warts in the skin. The varicella-zoster virus
is a DNA virus. The parvovirus group includes DNA viruses. Erythema infectiosum is caused by parvovirus B19.
98. A 57-year-old male patient presents with weight loss, myalgias, and tender purpuric papules and nodules on the
extremities. A biopsy reveals a necrotizing medium-vessel vasculitis. Which other clinical finding is likely to be
found in this patient?
A. Alopecia
B. Hearing loss
C. Hepatitis B surface antigen positivity
D. Normal ESR and C-protein
E. c-ANCA positivity
ANSWER: C.
EXPLANATIONS:
C. TRUE: Hepatitis B infection has been strongly associated with PAN development, typically within the first 6
months of infection. About 8 % of PAN cases have associated hepatitis B.
A, B, D, and E. FALSE: The American College of Rheumatology (ACR) has developed diagnostic criteria to
differentiate polyarteritis nodosa.
99. A 2-year-old male presents to the emergency room with painful oral ulcers and difficulty swallowing. There is
no evidence of other mucocutaneous lesions. The family denies recent upper respiratory illness. The patient was
given a few doses of acetaminophen for pain and diphenhydramine for sleep. What is the most likely diagnosis?
A. Aphthous ulcers
B. Double-stranded DNA viral infection
C. Single-stranded RNA viral infection
D. Stevens-Johnson syndrome
E. Medication-induced mucositis
45
ANSWER: B.
EXPLANATIONS:
B. TRUE: This is a case of herpes gingivostomatitis primarily caused by HSV-1 infection. Vesicles and ulcers can
present anywhere in the oral cavity, lips, and perioral skin. The gingiva is typically edematous and erythematous,
as seen in the photo. Herpes simplex virus is a double-stranded DNA virus.
A. FALSE: Ulcers can present anywhere in the oral cavity, lips, and perioral skin.
C. FALSE: ssRNA viral infection would imply that this is a case of herpangina/hand-foot-mouth disease, however,
extensive perioral involvement and lack of palmoplantar lesions make this unlikely.
D. FALSE: There have been multiple reports of acetaminophen-induced SJS, however, extensive perioral
involvement, rare occurrence, and lack of other mucosal involvement make this unlikely.
E. FALSE: Chemotherapeutic agents are the most likely cause of medication-induced mucositis.
100. When do the majority of neonatal herpes simplex virus (HSV) infections take place?
A. Perinatally
B. Gestation (in-utero)
C. Postnatally
D. During breastfeeding
E. Equal incidence
ANSWER: A.
EXPLANATIONS:
A. TRUE: Most neonates (80-90%) with confirmed HSV infection acquire the virus following contact with the
infectious genital mucous membrane secretions or herpetic lesions themselves, if present at the time of delivery.
B and C. FALSE: Acquisition of HSV infection can happen in utero or immediately after birth but occurs far less
commonly than perinatal infections at 4% and 10%, respectively.
D. FALSE: HSV can be acquired during breastfeeding but requires contact with lesions of HSV. Latent HSV and
recurrent HSV infections are not a contraindication to breastfeeding an infant. The only instance in which HSV
infections would be a contraindication is if there are lesions at the site of breastfeeding.
E. FALSE: Rates of HSV infection are much lower postnatally and in-utero.
101. A patient presents with the lesion shown below. Which of the following drugs should be given as post-exposure
prophylaxis (PEP) for this disease?
A. Penicillin
B. Doxycycline
C. Raxibacumab
D. Rifampin
46
Correct Answer: B. Doxycycline
EXPLANATIONS:
The black eschar depicted is due to cutaneous anthrax. Anthrax is a zoonotic infection caused by Bacillus anthracis.
Most anthrax cases are cutaneous (95%); the remaining cases are inhalational and gastrointestinal. The anthrax ulcer
and surrounding edema evolve into a black eschar within 7-10 days and last for 7-14 days before separating and
leaving a permanent scar. The Centers for Disease Control and Prevention (CDC) have issued updated guidelines on
anthrax post-exposure prophylaxis (PEP) and treatment in nonpregnant and pregnant adults. The CDC recommends
that all individuals exposed to aerosolized B. anthracis spores should receive a full 60 days of PEP antimicrobial drugs,
regardless of their vaccination status. Before October 2001, the first-line treatment for anthrax infection and
prophylaxis was penicillin; however, this is not the case for bioterrorism-related cases because of the concern
regarding genetically engineered penicillin-resistant anthrax strains. Thus, the CDC now recommends ciprofloxacin or
doxycycline.
Incorrect Answers:
A. The preferred agent used to treat non-bioterrorist anthrax is penicillin.
C. Raxibacumab, a monoclonal antibody directed at the protective antigen of B. anthracis, is available from the CDC
to treat inhalational anthrax in adults and children.
D. Rifampin can be used as part of a multi-drug regimen for inhalational anthrax, but it is not used as post-exposure
prophylaxis or as a single agent. It is also employed as part of a multidrug regimen for the treatment of tuberculosis.
102. The clinician receives an alert in her inbox informing her that a culture taken from a 2-day-old newborn with
a suspected cytomegalovirus (CMV) infection came back positive. The child has no cutaneous signs of infection,
and a systemic workup had been performed as a result of the mother’s diagnosis of CMV at 7 months of
pregnancy. Which of the following is the most appropriate next step in management?
A. Parenteral ganciclovir
B. Parenteral acyclovir
C. Oral valganciclovir
D. No treatment
E. Parenteral ganciclovir or oral valganciclovir
ANSWER: E.
EXPLANATIONS:
E. TRUE: This patient presented with no signs or symptoms of congenital or perinatally acquired CMV. The culture
could have been taken from the birth fluid, placenta, or the umbilical cord in the absence of cutaneous findings in
the newborn. Despite the lack of findings, 10-15% of children with asymptomatic CMV infection are still at risk for
developing sensorineural hearing loss. Treatment is advised for culture-positive CMV. Of note, cultures that are
positive for CMV beyond 1-2 weeks of life are not able to delineate congenital and perinatal infection. Treatment
consists of the nucleosides analogs ganciclovir and valganciclovir, with the former requiring parenteral
administration. Acyclovir is used in the treatment and management of herpes simplex virus.
103. An 8-year-old male presents with crusty erythematous plaques (see photo). The patient feels well otherwise.
Which of the following is incorrect regarding this condition?
A. It is caused by superficial gram-positive organisms.
B. Pediculosis capitis is a risk factor for the
development of this condition.
C. These infections are sometimes followed by acute
glomerulonephritis.
D. Topical antibiotic monotherapy is appropriate.
E. Topical polymyxin is an effective therapy.
47
ANSWER: E.
EXPLANATIONS:
E. TRUE: The figure displays the clinical appearance of impetigo. Polymyxin covers gram-negative infections and would
be ineffective in treating impetigo.
A. FALSE: Impetigo is most commonly caused by superficial staphylococcal and streptococcal skin infections.
B. FALSE: Pediculosis capitis (head lice infestation) can be complicated by impetiginization due to excessive scratching.
C. FALSE: Multiple strains of streptococcus (type 49, 55, 57; 60 strains; and strain M-type 2) lead to an immune
complex disorder causing poststreptococcal glomerulonephritis.
D. FALSE: For localized lesions, topical antibiotics such as mupirocin have been effective.
104. What of the following is incorrect regarding the condition exhibited in this photo?
ANSWER: B.
EXPLANATIONS:
B. TRUE: The figure displays the clincal appearance of green nail syndrome (pseudomonal paronychia). Prolonged
water exposure is a risk factor for developing green nail syndrome
A. FALSE: Green nail syndrome is characterized by distal onycholysis and a greenish discoloration in multiloculated
areas.
C. FALSE: Effective treatment of the paronychia should lead to complete resolution and a normal-appearing nail
within the next several months.
D. FALSE: Trimming of the nails and topical application of triple antibiotic ointment have been effective.
E. FALSE: Occlusion and moisture of plantar feet (i.e. casting and water seepage) have led to a similar discoloration
of the soles.
ANSWER: A.
EXPLANATIONS:
A. TRUE: This is the false statement. T. brucei gambiense is the West African form, and T. brucei rhodesiense is the
East African form.
B. FALSE: T. cruzi is the South and Central American form.
C. FALSE: The tsetse fly is the disease vector in Africa.
D. FALSE: Winterbottom's sign is a symptom of the African form. It consists of posterior cervical lymphadenopathy.
E. FALSE: Romana's sign may be part of the South American form. It consists of unilateral eyelid edema and
conjunctivitis.
48
106. A neonate is born to a mother with suspected cytomegalovirus infection during pregnancy. The neonate
demonstrates microcephaly, pneumonitis, hepatosplenomegaly, and cutaneous signs of extramedullary
hematopoiesis. Which of the following scenarios most likely represents the causative factor contributing to these
findings?
A. The neonate was born to a mother who had a viral reactivation of cytomegalovirus and was
asymptomatic.
B. The neonate was born to a mother who had a viral reactivation of cytomegalovirus and was
symptomatic.
C. The neonate was born to a mother who had a primary viral infection in her third trimester.
D. The neonate was born to a mother who had a primary viral infection in her first trimester.
E. The neonate acquired the infection perinatally following exposure to infectious secretions in
the birth canal.
ANSWER: D.
EXPLANATIONS:
D. TRUE: Cytomegalovirus represents the most commonly diagnosed congenital infection, and its incidence is as
high as 1-2% of all births. Cutaneous findings include those seen with additional viral infections causing “blueberry
muffin” syndrome, which is due to extramedullary hematopoiesis. Systemic findings of congenital CMV infection
are outlined in the above vignette but can also include chorioretinitis. Primary CMV infection during the first
trimester carries the greatest risk of cutaneous and systemic involvement.
A and B. FALSE: Most newborns from mothers who experienced viral reactivation of CMV during pregnancy are
normal and present without the signs of congenital infections.
C. FALSE: While primary infection during the third trimester certainly carries a greater risk of symptomatic
congenital CMV infection, the cessation of organogenesis and development by this time serves as a protective
factor in contrast to the first trimester.
E. FALSE: CMV infection can be acquired perinatally following infectious exposure to secretions during vaginal
birth, but this is a much lower risk factor for acquiring long-term sequelae. It is important to note that 10-15% of
children may be born with asymptomatic CMV infection and are still at risk for sensorineural hearing loss.
107. Which of the following tickborne diseases is matched improperly with its corresponding tick vector?
A. Lyme disease - Ixodes scapularis
B. Ehrlichiosis - Ambylomma americanum
C. Southern tick-associated rash illness (STARI) - Ambylomma americanum
D. Tularemia - Rhipicephalus sangunineus
E. Babesiosis - Ixodes scapularis
ANSWER: D.
EXPLANATIONS:
D. TRUE: Tularemia is caused by Francisella tularensis. It is spread by Dermacentor variabilis, Dermacentor andersoni,
and Amblyomma americanum. Rhipicephalus sangunineus is known to spread Rocky Mountain spotted fever.
A. FALSE: Lyme disease is caused by Borrelia burgdorferi and transmitted by Ixodes scapularis.
B. FALSE: Ehrlichiosis is caused by Ehrlichia chaffeensis transmitted by Ambylomma americanum.
C. FALSE: STARI is caused by an unknown agent transmitted by Ambylomma americanum .
E. FALSE: Babesiosis caused by Babesia microti is spread by Ixodes scapularis.
49